Audit evidence is obtained by the auditor when performing all of the following except when

2.Ratio estimate: $25000 (audit)/$27500(book) *550,000=500,000 (point estimate)

3. Difference Estimation: ($27,500-$25,000)/100*$2,000=$50,000 (adjustment required)

A deficiency in internal control exists when misstatements of the financial statements may occur and not be prevented, detected, or corrected on a timely basis by

A.    Outside consultants who issue a special-purpose report on internal control

B.    Management or employees in the normal course of performing their assigned functions

C.    Management when reviewing interim financial statements and reconciling account balances

D.    An independent auditor during the testing of controls phase of the consideration of internal control

B.

A deficiency in internal control exists when the design or operation of a control does not allow man­agement or employees, in the normal course of performing their assigned functions, to prevent, or detect and correct, misstatements on a timely basis.

A dual-purpose test

A.    Involves combining a risk assessment procedure and a test of controls

B.    Is performed when more persuasive audit evidence is needed

C.    Is designed to test both the efficiency and the reliability of a control

D.    Is designed and evaluated by considering each purpose of the test separately

D.

A dual-purpose test is designed and evaluated by considering each purpose of the test separately. For example, the auditor may design and evaluate the results of a test to examine an invoice to determine whether it has been approved and to provide substantive audit evidence of a transaction. Regarding incorrect answer a., a dual-purpose test involves performing a test of details (not a risk assessment procedure) and a test of controls on the same transaction concurrently. Regarding incorrect answer b., the evidence obtained is not more persuasive. Although it can be a more efficient audit technique if properly planned, the purpose of each test is considered separately.

Incorrect answer c. is a false statement.

A dual-purpose test

A.    Involves combining a risk assessment procedure and a test of controls

B.    Is performed when more persuasive audit evidence is needed

C.    Is designed to test both the efficiency and the reliability of a control

D.    Is designed and evaluated by considering each purpose of the test separately

D.

A dual-purpose test is designed and evaluated by considering each purpose of the test separately. For example, the auditor may design and evaluate the results of a test to examine an invoice to determine whether it has been approved and to provide substantive audit evidence of a transaction. Regarding incorrect answer a., a dual-purpose test involves performing a test of details (not a risk assessment procedure) and a test of controls on the same transaction concurrently. Regarding incorrect answer b., the evidence obtained is not more persuasive. Although it can be a more efficient audit technique if properly planned, the purpose of each test is considered separately.

Incorrect answer c. is a false statement.

A group engagement team of a nonissuer should ask a component auditor to communicate whether it complied with

A.    Ethical requirements relevant to the group audit.

B.    State licensure requirements applicable to all group locations.

C.    Generally accepted accounting principles relevant to the group audit.

D.    Contract billing requirements related to the group audit.

The correct answer is (A).

Regardless of whether reference will be made in the auditor's report on the group financial statements to the audit of a component auditor, the group engagement team should obtain an understanding of whether a component auditor understands and will comply with the ethical requirements that are relevant to the group audit and, in particular, is independent. Hence, the group engagement team should request a component auditor to communicate matters such as:

Whether the component auditor has complied with ethical requirements relevant to the group audit, including independence and professional competence (权限)

Identification of the financial information of the component on which the component auditor is reporting

The component auditor's overall findings, conclusions, or opinion

A management's specialist is best described as

A.    An individual or organization possessing expertise in a field other than accounting or auditing, whose work in that field is used by the entity to assist the entity in preparing the financial statements

B.    An individual or organization possessing expertise in a field other than accounting or auditing, whose work in that field is used by the auditor to assist the entity in preparing the financial statements

C.    A specialist engaged by management to assist the auditor in obtaining sufficient appropriate audit evidence

D.    An individual or organization possessing expertise in a field other than accounting or auditing, whose work in that field or whose data is used by the entity to assist the entity in preparing the financial statements

The correct answer is (A).

GAAS defines a management's specialist as an individual or organization possessing expertise in a field other than accounting or auditing, whose work in that field is used by the entity to assist the entity in preparing the financial statements.

The work of the individual or organization is used by the entity, not the auditor, to assist the entity in preparing the financial statements.

A specialist engaged by the auditor, not management, to assist the auditor in obtaining sufficient appropriate audit evidence is an auditor's specialist, not a management's specialist.

When the entity does the work using data provided by an individual or organization, it is not considered to be the use of a management's specialist by the entity.

Note the differences between A, B, and D. The three options are different:

  • A: Work in the field is used by the entity
  • B: Work in the field is used by the auditor
  • D: Work in the field or whose data is used by the entity

A member of the International Federation of Accountants (IFAC) may practice in a jurisdiction or belong to another professional organization that has less stringent requirements than the IESBA Code of Ethics for Professional Accountants (Code). Which set of requirements should the member follow under these circumstances? A. A member of IFAC shall not apply less stringent standards from those stated in the code under any circumstances. B. A member of IFAC shall not apply less stringent standards from those stated in the code unless prohibited from complying by law or regulation. C. A member of IFAC is allowed to comply with the less stringent requirements if these are imposed by an organization recognized by IFAC. D. A member of IFAC shall request a ruling from IESBA regarding a conflict between its standards and those of another organization.

B. A member of IFAC shall not apply less stringent standards from those stated in the code unless prohibited from complying by law or regulation. In this situation, the member shall comply with all other parts of the Code. Editor’s note: When in doubt, always follow the more “stricter” standards.

A senior auditor conducted a dual-purpose test on a client's invoice to determine whether the invoice was approved and to ascertain the amount and other terms of the invoice. Which of the following lists two tests that the auditor performed?

A.    Substantive procedures and analytical procedures.

B.    Substantive analytical procedures and tests of controls.

C.    Tests of controls and tests of details.

D.    Tests of details and substantive procedures.

The correct answer is (C).

A senior auditor conducted a dual-purpose test on a client's invoice to determine whether the invoice was approved and to ascertain the amount and other terms of the invoice. He/she concurrently performed tests of controls and tests of details.

Tests of controls - Checks performed to verify whether internal controls are working and are strong.

Tests of details - Substantive procedures that verify different assertions in the financial statements - E.g., Inspection, Existence/Occurrence, etc.

(A) and (D) are incorrect because they do not refer to a test of controls.

(B) is incorrect because verifying dollar amount of the invoice is a test of detail and not an analytical procedure.

A service organization provides processing services for a client’s sales orders. Which of the following information is relevant when gathering data for the report on the service organization’s internal controls?

A.    The client’s sales manager reviews accounts receivable balances.

B.    The client’s data entry clerk used the sales manager’s password to make unauthorized changes to customer prices.

C.    Credit limits are established and updated by the client’s credit department.

D.    The service organization’s system calculates accounts receivable balances.

D.

The correct answer is (D).

A service organization provides processing services for a client’s sales orders. Information most relevant when gathering data for the report on internal controls is the accounts receivable balances as the service organization provides processing services for a client’s sales orders.

Information related to the client's sales manager review of accounts receivable, information related to the client's data clerk making unauthorized changes to customer prices, or information related to the client's credit department and their development of credit limits would not provide information about the service organization's internal controls.

A significant risk A. Should be communicated to those charged with governance B. Requires special audit consideration C. Should be identified by considering the related controls that mitigate its potential effects D. Is related to fraud

B A significant risk is an identified and assessed risk of material misstatement that, in the auditor's professional judgment, requires special audit consideration. The auditor is required to communicate with those charged with governance an overview of the planned scope and timing of the audit and this may include how the auditor proposes to address the significant risks, but it is not required. In exercising judgment as to whether a risk is significant, the auditor should exclude the effects of identified controls related to the risk. One of the factors the auditor considers in identifying significant risks is whether the risk is a risk of fraud, but not all significant risks are fraud risks.

A weakness in internal control over recording retirements of equipment may cause an auditor to

A.    Trace additions to the "other assets" account to search for equipment that is still on hand but no longer being used.

B.    Select certain items of equipment from the accounting records and locate them in the plant.

C.    Review the subsidiary ledger to ascertain whether depreciation was taken on each item of equipment during the year.

D.    Inspect certain items of equipment in the plant and trace those items to the accounting records.

B.

The auditor may test controls over the recording of retirements by tracing certain items of equipment from the accounting records and locating them in the plant to make sure that they have not been retired. Additions to the "other assets" account should have nothing to do with the failure to record retirements of equipment. Tracing from the plant assets to the books would not locate assets which appear on the books even though they have been retired because none of the equipment traced would be retired equipment. If retired equipment that was not fully depreciated was not removed from the records, the related depreciation calculation would not reveal its retirement; further, fully depreciated equipment may or may not still be in use.

According to the profession's standards, which of the following is not required of a CPA performing a consulting engagement? A. Complying with Statements on Standards for Consulting Services B. Obtaining an understanding of the nature, scope, and limitations of the engagement C. Supervising staff who are assigned to the engagement D. Maintaining independence from the client

D. ET 201 requires a CPA performing a consulting engagement to comply with Standards on Consulting Services and to supervise staff assigned to the engagement. ET 202 requires that the CPA obtain an understanding of the nature, scope, and limitations of the engagement. ET 202 also states that performing a consulting service does not, of itself, impair independence.

According to US GAAS, all of the following are conditions that should be met to allow reference to the audit of a component auditor in the audit report on the group financial statements except

A.    The component auditor has performed an audit of the financial statements of the component in accordance with the relevant requirements.

B.    The component auditor has issued an audit report that is not restricted as to use.

C.    If the component’s financial statements are prepared using a different financial reporting framework from that used for the group financial statements, the measurement, recognition, presentation, and disclosure criteria that are applicable to all material items are similar.

D.    If the component’s financial statements are prepared using a different financial reporting framework from that used for the group financial statements, the conversion adjustments needed are not material.

The correct answer is (D).

If the Group Engagement Auditor decides not to assume responsibility for the Component Auditor’s work, the reference to the Component Auditor divides responsibility for the engagement among the Group Engagement Auditor and Component Auditor(s). The Group Engagement Auditor needs to ensure that:

The Component Auditor has audited the component per GAAS (or PCAOB Auditing Standards as applicable) i.e. in accordance with relevant requirements.

The Component Auditor’s report is not restricted as to use 

If the financial statements are prepared using a different financial reporting framework from that used for the group financial statements, the group engagement team has ensured the measurement, recognition, presentation, and disclosure criteria that are applicable to all material items are similar and obtained sufficient appropriate audit evidence for purposes of evaluating the appropriateness of the adjustments to convert the component’s financial statements to the financial reporting framework.

Even if the conversion adjustments needed to prepare are material as long as the auditor has applied audit procedures and obtained sufficient appropriate audit evidence, reference to the component auditor can be made.

According to US GAAS, all of the following matters should be communicated by the group engagement team to those charged with governance of the group except

A.    An overview of the nature of the group engagement team’s planned involvement in the work to be performed by the component auditors on the financial information of significant components

B.    Any limitations on the group audit

C.    Instances in which the group engagement team's evaluation of the work of a component auditor gave rise to a concern about the quality of that auditor's work

D.    All internal control deficiencies that are relevant to the group

The correct answer is (D).

The group engagement team should communicate to group management and those charged with governance of the group material weaknesses and significant deficiencies in internal control that are relevant to the group; not all internal control deficiencies. In addition to the other answer alternatives, the group engagement team should also communicate the following matters to those charged with governance of the group:

(1) an overview of the type of work to be performed on the financial information of the components, including the basis for the decision to make reference to the audit of a component auditor in the audit report on the group financial statements and
(2) fraud or suspected fraud involving group management, component management, employees who have significant roles in group-wide controls, or others in which a material misstatement of the group financial statements has or may have resulted from fraud.

According to US GAAS, all of the following statements about interpretive publications are true, except

A.    Interpretive publications are not auditing standards; they are recommendations on the application of US GAAS in specific circumstances.

B.    An auditor is not required to consider applicable interpretive publications in planning and performing an audit.

C.    Interpretive publications are issued under the authority of the Auditing Standards Board (ASB).

D.    Auditing interpretations of US GAAS are included in the codification of US GAAS, following the related AU section.

B.

An auditor is required to consider [should consider] applicable interpretive publications in planning and performing an audit. The other answer alternatives are all true statements.

Editor Note:  Regarding answer A., interpretive publications are not auditing standards. They are recommendations on the application of US GAAS in specific circumstances, including engagements for entities in specialized industries.

Regarding answer C., an interpretive publication is issued under the authority of the ASB after all ASB members have been provided an opportunity to consider and comment on whether it is consistent with US GAAS.

Regarding answer D., although interpretive publications are not auditing standards, auditing interpretations of US GAAS, a type of interpretive publication, are included in the codification of US GAAS. Auditing interpretations of US GAAS immediately follow the related AU section and have been assigned the same section number preceded by the number 9. (Not all AU sections have auditing interpretations of US GAAS.) Interpretive publications include: auditing interpretations of US GAAS (included in the codification of US GAAS following the related AU section); exhibits to US GAAS (included in the codification of US GAAS following the application and explanatory materials portion of AU sections); and auditing guidance in AICPA Audit and Accounting Guides and AICPA Auditing Statements of Position.

According to US GAAS, audit procedures performed on the consolidation process should include all of the following except

A.    Evaluating whether all components have been included in the group financial statements

B.    Evaluating the appropriateness, completeness, and accuracy of consolidation adjustments and reclassifications

C.    Perform procedures to identify material instances of non-compliance

D.    Evaluating whether any fraud risk factors or indicators of possible management bias exist

C.

The correct answer is (C).

The group engagement team should perform the following audit procedures on the consolidation process:

  • Evaluating whether all components have been included in the group financial statements.
  • Evaluate the appropriateness, completeness, and accuracy of consolidation adjustments and reclassifications and should evaluate whether any fraud risk factors or indicators of possible management bias exist.
  • If the financial information of a component has not been prepared in accordance with the same accounting policies applied to the group financial statements, the group engagement team should evaluate whether the financial information of that component has been appropriately adjusted for purposes of the preparation and fair presentation of the group financial statements.
  • If the group financial statements include the financial statements of a component with a financial reporting period-end that differs from that of the group, the group engagement team should evaluate whether appropriate adjustments have been made to those financial statements in accordance with the applicable financial reporting framework.

The auditor is not specifically required to perform procedures to identify instances of Non-Compliance in GAAS audits.

According to US GAAS, if the group engagement partner decides to assume (承担) responsibility for the work of a component auditor, the group engagement partner

A.    Should make reference to the audit of the component auditor in the audit report on the group financial statements

B.    Should be involved in the work of the component auditor, insofar as that work relates to the expression of an opinion on the group financial statements

C.    Need not assess the component auditor’s independence or professional competence

D.    Should make reference to the audit of the component auditor as well as name the component auditor in the audit report on the group financial statements

B.

If the group engagement partner decides to assume responsibility for the work of a component auditor, the group engagement partner is required to be involved in the work of the component auditor, insofar as that work relates to the expression of an opinion on the group financial statements.

Editor’s note: Remember, if as the group auditor, if you’re planning on assuming responsibility for a component auditor’s work, you want to take credit for it, and the best way to do so is to get involved with the component auditor’s work so make sure that a good job is being done!

Regarding incorrect answers A. and D., under these circumstances, no reference will be made to the component auditor in the group audit report. Further regarding incorrect answer D., if the group engagement auditor does not assume responsi­bility and thus, makes reference to the component auditor, the component auditor may also be named, but this is not a requirement. (Naming should only be done with the component auditor’s express permission and the component auditor’s report should be presented together with that of the audit report on the group financial statements.)

Regarding incorrect answer C., regardless of whether reference to a component auditor will be made in the audit report, the group engagement team should obtain an understanding of the component auditor which includes determining whether the component auditor understands and will comply with the ethical require­ments that are relevant to the group audit and, in particular, is independent. An assessment of the component auditor’s professional competence should also be performed.

There are other required elements of this under­standing: (1) the extent, if any, to which the group engagement team will be able to be involved in the work of the component auditor; (2) whether the group engagement team will be able to obtain information affecting the consolidation process from a component auditor; and (3) whether a component auditor operates in a regulatory environment that actively oversees auditors.

According to US GAAS, in general, misstatements in the financial statements, including omissions, are considered to be material when, individually or in the aggregate,

A.    If uncorrected, would preclude an unmodified opinion on the financial statements.

B.    If uncorrected, would require an adverse opinion on the financial statements.

C.    They could reasonably be expected to influence the economic decisions of users that are taken based on the financial statements.

D.    They are inconsistent with the criteria of the applicable financial reporting framework.

C.

In general, misstatements, including omissions, are considered to be material if, individually or in the aggregate, they could reasonably be expected to influence the economic decisions of users that are taken based on the financial statements. The concept of materiality is applied by the auditor when both planning and performing the audit; and in evaluating the effect of identified misstatements on the audit and uncorrected mis­statements, if any, on the financial statements. Judgments about materiality are made in light of surrounding cir­cumstances and involve both qualitative and quantitative considerations. These judgments are affected by the auditor’s perception of the financial information needs of users of the financial statements and by the size and/or nature of a misstatement.

Regarding incorrect answer D., it does not reflect the comprehensive nature of the considerations involved described in the preceding explanation of the correct answer.

Regarding incorrect answers A. and B., they are not true because US GAAS does not preclude nor require a type of opinion based solely on materiality—all modified opinions are due to matters that are judged by the auditor to be material.

According to US GAAS, regarding materiality considerations in group audits

A.    The group engagement team should set materiality for the group financial statements as a whole equal to that of the component with the lowest level when establishing the overall group audit strategy.

B.    Component materiality should be determined taking into account only those components referenced in the audit report on the group financial statements.

C.    A threshold (标准) for misstatements is determined in addition to component materiality.

D.    The group engagement team should review the component materiality determined by each component.

C.

A threshold for misstatements is determined in addition to component materiality. This is a threshold above which misstatements cannot be regarded as clearly trivial to the group financial statements. (Misstate­ments identified in the financial information of the component that are above this threshold for misstatements of the group are communicated to the group engagement team.) The group engagement team should determine materiality, including performance materiality, for the group financial statements as a whole when establishing the overall group audit strategy. Regarding incorrect answer A., there is no such requirement to set materiality equal to the component with the lowest level. Regarding incorrect answer B., component materiality should be determined taking into account all components, regardless of whether reference is made in the audit report on the group financial statements to the audit of a component auditor. Regarding incorrect answer D., component materiality is the materiality for a component that is determined by the group engagement team for the purposes of the group audit.

Editor’s note: Again, don’t let the term “group” or “component” throw you off on these types of questions; as the auditor you’re still performing the same planning procedures, which include materiality and the related threshold for clearly trivial items. One of the major differences with the group and component audits is considering the group and the component(s) as standalone entities for purposes of materiality.

According to US GAAS, the applicable financial reporting framework adopted by an entity may encompass financial accounting standards promulgated by a standards-setting organization; legislative or regulatory requirements; and/or other sources. These sources may also provide direction on the application of the applicable financial reporting framework. When conflicts in the guidance among these sources exist

A.    The source with the highest authority prevails.

B.    The auditor exercises professional judgment to determine the most appropriate guidance to advocate.

C.    The source that is the most prevalent in the entity’s industry should be followed.

D.    Decisions about an entity’s applicable financial reporting framework are the responsibility of management and, when appropriate, those charged with governance.

A.

When conflicts in the guidance among the sources of an entity’s applicable financial reporting framework exist, the source with the highest authority prevails.

Regarding incorrect answer B., it is a true statement, but it is not the best answer. Note that professional judgment is not to be used as the justification for decisions that are not otherwise supported by the facts and circumstances of the engagement or by sufficient appropriate audit evidence.

Regarding incorrect answer C., prevalent general and industry practices may be considered, but they would only prevail in a conflict if they were the source with the highest authority available on a particular issue.

Regarding incorrect answer D., it is a true statement, but it is not the best answer, i.e., it is not responsive to the question. As for the statement, management and, when appropriate, those charged with governance, have responsibility for the preparation and fair presentation of the financial statements in accordance with the applicable financial reporting framework. And the auditor evaluates management’s judgments in applying the entity’s applicable financial reporting framework.

According to US GAAS, to express an opinion, the auditor obtains reasonable assurance about whether the financial statements are free from material misstatement. Obtaining reasonable assurance directly relates to all of the following except

A.    Planning the work and properly supervising any assistants

B.    Recognizing that circumstances may exist that cause the financial statements to be materially misstated

C.    Determining and applying appropriate materiality level(s) throughout the audit

D.    Identifying and assessing risks of material misstatement (RMM)

B.

Answer B., recognizing that circumstances may exist that cause the financial statements to be mate­rially misstated, is not one of the actions the auditor takes. According to US GAAS, to obtain reasonable assurance, it is the reason that an auditor should plan and perform an audit with professional skepticism. The other answer alternatives are three of the four actions the auditor takes, according to US GAAS, to obtain reasonable assurance about whether the financial statements as a whole are free from material misstatement, whether due to fraud or error. The remaining action the auditor takes to obtain reasonable assurance is obtaining sufficient appropriate audit evidence [about whether material misstatements exist, through designing and implementing appropriate responses to the assessed risks].

According to US GAAS, when performing an audit on the financial information of a component, the group engagement team or the component auditor should perform procedures designed to identify relevant subsequent events at that component that occur between the date of the financial information of the component and

A.    The date when sufficient appropriate evidence on which to base the auditor’s opinion on the component financial information has been obtained

B.    The date of the audit report on the group financial statements

C.    The date of the release of the audit report on the group financial statements

D.    As near as practicable to the date of the release of the audit report on the group financial statements

B.

When the group engagement team or component auditors perform audits on the financial infor­mation of components, the group engagement team or the component auditors should perform procedures designed to identify events at those components that occur between the dates of the financial information of the components and the date of the audit report on the group financial statements. In addition, that may require adjustment to, or disclosure in, the group financial statements.

According to US GAAS, when the auditor of the group financial statements is assuming responsibility for the work of component auditors, for components that are not significant components, the group engagement team should perform

A.    Analytical procedures at the group level

B.    An audit using component materiality

C.    An audit of one or more account balances, classes of transactions, or disclosures relating to the likely significant risks of material misstatement of the group financial statements

D.    Specified audit procedures relating to the likely significant risks of material misstatement of the group financial statements

A.

When the auditor of the group financial statements is assuming responsibility for the work of compo­nent auditors, for components that are not significant components, the group engagement team should perform analytical procedures at the group level. The group engagement team, or a component auditor on its behalf, should perform one or more of the other answer alternatives for a component that is significant not due to its individual financial significance but because it is likely to include significant risks of material misstatement of the group financial statements due to its specific nature or circumstances.

Editor’s note: Consider the language in the question; the group auditor is assuming responsibility for the work of component auditors. For components that the component auditors have not audited, why should the group auditor perform audit work in addition to what they’ve agreed with the entity on (i.e. perform audits in addition to the group)? Considering this logic will help you answer this question from a logic perspective.

According to US GAAS, when the auditor of the group financial statements is assuming responsibility for the work of component auditors, for a component that is significant due to its individual financial significance to the group, the group engagement team, or a component auditor on its behalf, should perform

A.    Analytical procedures at the group level

B.    An audit using component materiality

C.    An audit of one or more account balances, classes of transactions, or disclosures relating to the likely significant risks of material misstatement of the group financial statements

D.    Specified audit procedures relating to the likely significant risks of material misstatement of the group financial statements

B.

For a component that is significant due to its individual financial significance to the group, the group engagement team, or a component auditor on its behalf, should perform an audit of the financial information of the component, adapted as necessary to meet the needs of the group engagement team, using component materiality.

Regarding incorrect answer A., for components that are not significant components, the group engagement team should perform analytical procedures at the group level.

Regarding incorrect answers C. and D., these plus answer a. are the options, one or more of which, that should be performed for a component that is significant not due to its individual financial significance but because it is likely to include significant risks of mate­rial misstatement of the group financial statements due to its specific nature or circumstances

After obtaining an understanding of the entity and its environment and assessing the risk of material misstatement, an auditor decided to perform tests of controls. The auditor most likely decided that

A.    It would be efficient to perform tests of controls that would result in a reduction in planned substantive tests.

B.    Additional evidence to support a further reduction in control risk is not available.

C.    An increase in the assessed level of control risk is justified for certain financial statement assertions.

D.    There were many internal control weaknesses that could allow errors to enter the accounting system.

A.

Answer a., the auditor most likely decided that the internal control system was effective and thus, it would be efficient to perform tests of controls that would result in a reduction in planned substantive tests. Regarding incorrect answer b., if evidence is not available, tests of controls are not performed. Regarding incorrect answer c., as the assessed level of control risk increases, the auditor is less likely to test controls. Regarding incorrect answer d., if the auditor is aware of many internal control weaknesses, the assessed level of control risk will be high, and controls will not be tested.

Although financial reporting frameworks may discuss materiality in different terms, they generally explain all of the following except

A.    Misstatements, including omissions, are considered to be material if they, individually or in the aggregate, could reasonably be expected to influence the economic decisions of users made on the basis of the financial statements.

B.    Judgments about materiality are made in light of surrounding circumstances and are affected by the size or nature of a misstatement, or a combination of both.

C.    Judgments about matters that are material to users of the financial statements are based on a consideration of the common financial information needs of users as a group.

D.    The possible effect of misstatements on specific individual users is also considered.

D.

The possible effect of misstatements on specific individual users, whose needs may vary widely, is not considered.

An audit client failed to maintain copies of its procedures manuals and organizational flowcharts. What should the auditor do in an audit of financial statements?

A.    Issue a qualified opinion on the basis of a scope limitation

B.    Document the auditor’s understanding of internal controls.

C.    Assess control risk at the maximum level

D.    Restrict the auditor's responsibility to assess the effectiveness of controls in the audit engagement letter

B.

The auditor should document key elements of the auditor's understanding of internal control regard­less of the client’s records. The client’s failure to do so does not constitute a scope limitation; necessitate that the auditor assess control risk at the maximum level; nor restrict the auditor’s responsibility to assess the effec­tiveness of controls.

An auditor assesses the risk of material misstatement at the assertion level to A. Obtain an understanding of the entity’s environment B. Determine the auditor's materiality levels C. Determine further risk assessment procedures D. Determine further audit procedures

D. The risk of material misstatement (RMM) at the assertion level is assessed in order to determine the nature, extent, and timing of further audit procedures necessary to obtain sufficient appropriate audit evidence. This evidence enables the auditor to express an opinion on the financial statements at an acceptably low level of audit risk. Regarding incorrect answer A., the auditor performs risk assessment procedures to obtain an under­standing of the entity and its environment, including its internal control. This understanding aids the identification and assessment of the RMM; obtaining an understanding of the entity’s environment is not the result of the assessment of the RMM. Regarding incorrect answer B., judgments about materiality levels provide a basis for the assessment of the RMM as well as the determination of the nature and extent of risk assessment proce­dures and the nature, extent, and timing of further audit procedures. They are not a result of the assessment of the RMM. Regarding incorrect answer C., risk assessment procedures are also performed (in addition to obtain an understanding of the entity), to identify and assess the RMM; not vice versa.

An auditor has set the materiality level for the financial statements as a whole at $125,000. Which of the following misstatements would the auditor most likely consider material?

A.    The client did not record $47,000 in trade accounts payable at year-end.

B.    The client did not disclose $45,000 of related party transactions in the footnotes.

C.    The client misclassified $42,000 of supplies expense as a miscellaneous expense.

D.    The client's estimate of the allowance for doubtful accounts is $40,000 more than the auditor's estimate.

The correct answer is (B).

The materiality of the transaction to the financial statement users does not depend solely on the recorded amount of the transaction but also on other specific relevant factors, such as the nature of the related party relationship. Related party transactions may indicate an increased risk of material misstatement of the financial statements. Hence, amongst all undisclosed $45,000 would most likely be considered material by the auditor.

An auditor is concerned about a policy of management override as an inherent limitation of internal control. Which of the following tests would best assess the validity of the auditor’s concern?

A.    Matching purchase orders to accounts payable

B.    Verifying that approved spending limits are not exceeded

C.    Tracing sales orders to the revenue account

D.    Reviewing minutes of board meetings

B.

Verifying that approved spending limits are not exceeded would best assess the validity of the auditor’s concern about a policy of management override because it is management’s responsibility to authorize expenditures. Irregularities which might be discovered by matching purchase orders to accounts payable or tracing sales orders to revenue accounts might not involve management. Undetected management override of controls would not be described in minutes of board meetings.

An auditor is determining if internal control relative to the revenue cycle of a wholesaling entity is operat­ing effectively in minimizing the failure to prepare sales invoices. The auditor most likely would select a sample of transactions from the population represented by the

A.    Cash receipts file

B.    Shipping document file

C.    Customer order file

D.    Sales invoice file

B.

The auditor should make the selection from the shipping document file. Attempting to match ship­ping documents to invoices would reveal goods shipped that were not invoiced. The cash receipts file contains evidence of payments, so it would be highly unlikely to discover payments received from customers who weren’t billed. The customer order file would be a workable but inefficient choice by the auditor because normally goods are not invoiced until they are shipped, so it would involve checking shipping documents as well to confirm the sale. The sales invoice file would not be used because the auditor is looking for sales that were not invoiced.

An auditor may decide to perform only substantive procedures for specific assertions because the auditor believes

A.    Control policies and procedures are unlikely to pertain to the assertions.

B.    The entity's control environment, monitoring, and control activities are interrelated.

C.    Sufficient audit evidence to support the assertions is likely to be available.

D.    More emphasis on tests of controls than substantive tests is warranted.

A.

In some cases, the auditor may determine that performing only substantive procedures is appropriate for specific relevant assertions and risks. In those circumstances, the auditor may exclude the effect of controls from the relevant risk assessment. This may be because the auditor's risk assessment procedures have not identified any effective controls relevant to the assertion or because testing the operating effectiveness of controls would be inefficient.

An auditor searching for related-party transactions should obtain an understanding of each subsidiary’s relationship to the total entity because

A.    The business structure may be deliberately designed to obscure related-party transactions.

B.    Intercompany transactions may have been consummated on terms equivalent to arm’s-length transactions.

C.    This may reveal whether particular transactions would have taken place if the parties had not been related.

D.    This may permit the audit of intercompany account balances to be performed as of concurrent dates.

A.

When searching for related-party transactions, the auditor should obtain an understanding of each subsidiary’s relationship to the total entity because business structure and operating style are occasionally deli­berately designed to obscure related party transactions. Answers b., c., and d. are not reasons for an auditor to obtain an understanding of each subsidiary’s relationship to the total entity.

An auditor should design the written audit plan (or program) so that

A.    All material transactions will be selected for substantive testing.

B.    Substantive tests prior to the balance sheet date will be minimized.

C.    The audit procedures selected will achieve specific audit objectives.

D.    Each account balance will be tested under either tests of controls or tests of details.

C.

The auditor should prepare a written audit plan (or program). The audit plan should detail the nature, extent, and timing of the audit procedures that are necessary to accomplish the objectives of the audit. All material transactions* and all account balances are not required to be tested in all circumstances. Minimizing substantive tests prior to the balance sheet date is not required.

An auditor suspects that a client's cashier is misappropriating cash receipts for personal use by lapping customer checks received in the mail. In attempting to uncover this embezzlement scheme, the auditor most likely would compare the A. Dates checks are deposited per bank statements with the dates remittance credits are recorded B. Daily cash summaries with the sums of the cash receipts journal entries C. Individual bank deposit slips with the details of the monthly bank statements D. Dates uncollectible accounts are authorized to be written off with the dates the write-offs are actually recorded

A. Lapping involves the theft of one customer's payment and subsequently crediting the customer with payment made by another customer. Future remittances may be deposited but would be credited to the account from which funds were stolen, thus comparison of remittance dates would detect the scheme. Answers (b), (c), and (d) occur after the theft and would not show differences to pursue.

An auditor suspects that certain client employees are ordering merchandise for themselves over the internet without recording the purchase or receipt of the merchandise. When vendors' invoices arrive, one of the employees approves the invoices for payment. After the invoices are paid, the employee destroys the invoices and the related vouchers. In gathering evidence regarding the fraud, the auditor most likely would select items for testing from the file of all: A. Cash disbursements; B. Approved vouchers; C. Receiving reports; D. Vendors' invoices

A. Since the employee is destroying the invoices and related vouchers, the most obvious documentation remaining would be the file of all cash disbursements. The auditor would select items from this file and then attempt to trace from specific cash disbursements to the related invoices and approved vouchers. Missing documentation might be indicative of fraud. C is in correct. Selecting items from the file of receiving reports will not identify fraudulent purchases that are shipped directly to the employees' home addresses.

An auditor uses the assessed risk of material misstatement to A. Evaluate the effectiveness of the entity's internal control policies and procedures. B. Identify transactions and account balances where inherent risk is at the maximum. C. Indicate whether materiality thresholds for planning and evaluation purposes are sufficiently high. D. Determine the acceptable level of detection risk for financial statement assertions.

D. The auditor should design and perform further audit procedures whose nature, extent, and timing are responsive to the assessed risk of material misstatement (RMM) at the relevant assertion level. Detection risk is a function of the effectiveness of an auditing procedure and its application by the auditor; therefore, the assessed RMM will determine the acceptable level of detection risk and hence the appropriate audit procedures to be performed. The effectiveness of internal control, inherent risk, and materiality thresholds are all factors that help the auditor assess the RMM.

An auditor who identifies a potential fraud that is significant within the context of the audit under generally accepted government auditing standards would most appropriately respond first in which of the following manners? A. Recommend a separate engagement to determine whether fraud has occurred. B. Extend audit procedures as necessary to determine whether fraud has occurred. C. Refer investigation of the potential fraud to the party with oversight responsibility. D. Communicate the potential fraud directly to management.

B. The auditor's responses to address specifically identified risks of fraud may include changing the nature, timing, and extent of auditing procedures in the following ways The nature of auditing procedures performed may be changed to provide more reliable audit evidence. The timing of substantive tests may need to be modified and should be performed at or near the end of the reporting period. The extent of the procedures applied should be increased by increasing the sample size. Hence, the auditor will extend audit procedures as necessary to determine whether fraud has occurred

The Public Company Accounting Oversight Board (PCAOB) uses the term internal control over financial reporting to describe a process that does not include which of the following?

A.    Procedures performed by the auditor

B.    Procedures that pertain to the maintenance of reasonably detailed records that accurately and fairly reflect the transactions and dispositions of the company's assets

C.    Procedures that provide reasonable assurance that transactions are recorded as necessary to permit preparation of the financial statements in accordance with GAAP and that receipts and expenditures are made only in accordance with company management and director authorization

D.    Procedures that provide reasonable assurance regarding prevention or timely detection of unauthorized acquisition, use, or disposition of company assets that could have a material effect on the financial statements

A.

An auditor's procedures performed during either an audit of ICFR or an audit of financial statements are not part of a company's internal control over financial reporting (ICFR). The term internal control over financial reporting is defined as a process designed by, or under the supervision of, the company's principal executive and principal financial officers, or persons performing similar functions, and effected by the company's board of directors, management, and other personnel, to provide reasonable assurance regarding the reliability of financial reporting and the preparation of financial statements for external purposes in accordance with GAAP.

Answers B, C, and D describe what its policies and procedures pertain to or provide.

An entity's internal control requires for every check request that there be an approved voucher, supported by a prenumbered purchase order and a prenumbered receiving report. To determine whether checks are being issued for unauthorized expenditures, an auditor most likely would select items for testing from the population of all

A.    Purchase orders

B.    Canceled checks

C.    Receiving reports

D.    Approved vouchers

B.

When internal control dictates that each check be accompanied by an approved voucher, and supported by a prenumbered purchase order and a prenumbered receiving report, the auditor would select items for testing from the population of all the canceled checks. If the auditor were to consider populations made up of all purchase orders, receiving reports, or approved vouchers, those canceled checks which were issued without such supporting documentation would not be discovered.

As a result of control testing, a CPA has decided to reduce control risk. What is the impact on the substantive testing sample size if all other factors remain constant?

A.    The sample size would be irrelevant.

B.    The sample size would be higher.

C.    The sample size would be lower.

D.    The sample size would be unaffected.

The correct answer is (C).

A CPA has decided to reduce control risk, so the substantive testing sample size would be lower.

The CPA has reduced control risk and can afford a higher detection risk and will be satisfied with a smaller, less time-consuming sample size during substantive testing. With lower control risk, an auditor can afford a higher detection risk and use a smaller sample size.

As the acceptable level of detection risk decreases, an auditor may A. Reduce substantive testing by relying on the assessments of inherent risk and control risk. B. Postpone the planned timing of substantive tests from interim dates to the year-end. C. Eliminate the assessed level of inherent risk from consideration as a planning factor. D. Lower the assessed level of control risk from high to low.

B. A decrease in the acceptable level of audit risk or in the amount considered material will result in the auditor's modifying the audit plan to obtain greater assurance from substantive testing by (1) selecting a more effective audit procedure, (2) applying procedures nearer to year end, or (3) increasing the extent of particular tests.

As the result of tests of controls, an auditor assessed control risk too low and decreased substantive testing. This assessment occurred because the true deviation rate in the population was:

More than the deviation rate in the auditor's sample. If the actual deviation rate in the population exceeds the maximum deviation rate based on the sample, control risk will be understated, since the control will be less effective than sample results would indicate.

Attribute sampling is primarily used for testing internal controls (testing for specific characteristics [seeking IC errors]). Often the attribute sampling application can be identified by finding the option that deals with yes-or-no questions (e.g., is the invoice properly approved?)

Variables sampling and PPS sampling are typically used in substantive testing of account balances (Estimating the dollar value of the population)

Audit programs should be designed so that

A.    Most of the required procedures can be performed as interim work.

B.    Inherent risk is assessed at a sufficiently low level.

C.    The auditor can make constructive suggestions to management.

D.    The audit evidence gathered supports the auditor's conclusions.

D. 

The primary purpose of the audit is to provide users of the financial statements with an opinion. The objective of the auditor is to design and perform audit procedures that enable the auditor to obtain sufficient appropriate audit evidence to be able to draw reasonable conclusions on which to base the auditor’s opinion. The design of the audit program has no effect on inherent risk. Procedures may be performed prior to the balance sheet date only if the effectiveness of interim work is not likely to be impaired. Suggestions to management are secondary considerations in an audit.

Audit risk is a function of A. The risk of material misstatement and inherent risk B. Control risk and detection risk C. Inherent risk and control risk D. The risk of material misstatement and detection risk

D. Audit risk is the risk that the auditor expresses an inappropriate audit opinion when the financial statements are materially misstated. Audit risk is a function of the risk of material misstatement (RMM) and detection risk. The RMM is the risk that the financial statements are materially misstated prior to the audit. This consists of two components at the assertion level: (1) inherent risk, which is the susceptibility of an assertion about a class of transaction, account balance, or disclosure to a misstatement that could be material, either indi­vidually or when aggregated with other misstatements, before consideration of any related controls; and (2) con­trol risk, which is the risk that a misstatement that could occur in an assertion about a class of transaction, account balance, or disclosure and that could be material, either individually or when aggregated with other mis­statements, will not be prevented, or detected and corrected, on a timely basis by the entity’s internal control. Detection risk is the risk that the procedures performed by the auditor to reduce audit risk to an acceptably low level will not detect a misstatement that exists and that could be material, either individually or when aggregated with other misstatements. It is a function of the effectiveness of an audit procedure and of its application by the auditor, i.e., an auditor might select an inappropriate audit procedure, misapply an appropriate audit procedure, or misinterpret the audit results.

Auditors try to identify predictable relationships when using analytical procedures. Relationships involving transactions from which of the following accounts most likely would yield the highest level of evidence? A. Accounts receivable; B. Interest expense; C. Accounts payable; D. Tavel and entertainment expense.

B Relationships among income statement accounts tend to be more predictable than balance sheet accounts (AR, AP)because they present transactions over a period of time rather than at one point in time. In addition, relationships involving transactions subject to management discretion (travel and entertainment ) are less predictable.

Before applying principal substantive tests to an entity’s accounts receivable at an interim date, an auditor should

A.    Consider the likelihood of assessing the risk of incorrect rejection too low

B.    Project sampling risk at the maximum for tests covering the remaining period

C.    Ascertain that accounts receivable are immaterial to the financial statements

D.    Assess the difficulty in controlling the incremental audit risk

D.

When substantive procedures are performed at an interim date, the auditor should perform further substantive procedures or a combination of substantive procedures and tests of controls covering the remaining period to provide a reasonable basis for extending the audit conclusions to the end of the period. Interim testing increases the risk that misstatements existing at the period end will not be detected. Evaluations of in appro­priate assessments of risk would not be a factor in deciding whether interim testing would be appropriate. Sam­pling risk need not necessarily be projected at the maximum for tests covering the remaining period. Whether or not substantive tests of accounts receivable may be performed at an interim date does not hinge on whether the balance is material to the financial statements.

Business risk

A.    Is broader than the risk of material misstatement of the financial statements

B.    Does not include the risk of material misstatement of the financial statements

C.    Is management’s concern, not the auditor’s

D.    Gives rise to risks of material misstatement in all cases

A.

Business risk is broader than the risk of material misstatement of the financial statements, though it includes the latter. Usually, management identifies business risks and develops approaches to address them. (Such a risk assessment process is part of internal control.) The auditor should also consider business risk because an understanding of the business risks facing the entity increases the likelihood of identifying risks of material misstatement. This is because most business risks will eventually have financial consequences and, therefore, an effect on the financial statements. Not all business risks give rise to risks of material misstatement; thus, the auditor does not have a responsibility to identify or assess all business risks. (Whether a business risk may result in a risk of material misstatement is considered in light of the entity’s circumstances.)

Editor Note:  Business risk is defined by US GAAS as the risk resulting from significant conditions, events, circumstances, actions, or inactions that could adversely affect an entity’s ability to achieve its objectives and execute its strate­gies or from the setting of inappropriate objectives and strategies.

Deviation rate in the sample

Is the auditor's best estimate of the deviation rate in the population from which it was selected.

Each of the following is an ethical principle that should guide the work of auditors in the conduct of audits under government auditing standards, except A. Materiality B. Integrity C. The public interest D. Proper use of government information

A. Materiality is not an ethics principle. The AICPA Code of Professional Conduct outlines public interest, integrity, and confidentiality as principles. The IFAC Code of Ethics for Professional Accountants outlines public interest, integrity, and confidentiality as principles. Responsibility and confidentiality both encompass “proper use of government information.”

Equipment acquisitions that are misclassified as maintenance expense most likely would be detected by an internal control procedure that provides for

A.    Segregation of duties of employees in the accounts payable department

B.    Authorization by the board of directors of significant equipment acquisitions

C.    Investigation of variances within a formal budgeting system

D.    Independent verification of invoices for disbursements recorded as equipment acquisitions

C.

The investigation of variances within a formal budgeting system would identify any unusual and unanticipated fluctuations in the repair and maintenance accounts when asset acquisitions are recorded there incorrectly. The segregation of [incompatible] duties is a good control; however,

answer A. would not ensure that equipment acquisitions were not misclassified.

Answer B. would not prevent the recording of an acquisition to the repair and maintenance accounts, nor would it serve to identify misclassifications.

Answer D. would not ensure that equip­ment purchases were recorded properly because these invoices only represent those acquisitions which are already properly recorded as fixed assets.

Existence is generally a more relevant assertion than completeness when auditing the revenue cycle. The risk that accounts receivable and sales will be overstated (existence) is high, while the risk that accounts receivable and sales will be understated (completeness) is low.

For accounts payable, the completeness and accuracy assertions are generally more relevant than the existence and rights and obligations assertions, because the risk of understatement is greater than the risk of overstatement.

Factor Sample size Want less deviation More Accept more deviation Less

Factor Sample size Expect less deviation Less Expect more deviation More

Facts: Total book value $550,000 Audited book value $27,500 Audited item true value $25,000 Population 2000 items Item tested 100 items Audited items average value $250 Standard error of mean $10

1.Mean-per-Unit Estimation: $250*2,000=$500,000(point estimate); $10*2,000=+/-2,000 (at 1 standard deviation)precision

For effective internal control, the Accounts Payable Department generally should

A.    Stamp, perforate, or otherwise cancel supporting documentation after payment is mailed

B.    Ascertain that each requisition is approved as to price, quantity, and quality by an authorized employee

C.    Obliterate the quantity ordered on the Receiving Department copy of the purchase order

D.    Establish the agreement of the vendor's invoice with the receiving report and purchase order

D.

The agreement of the documents will verify that the goods were ordered (purchase order), received (receiving report), and the company has been billed (vendor’s invoice). The individual signing the checks, not accounts payable, should stamp, perforate, or otherwise cancel the supporting documentation. The purchasing department, not accounting, is involved with the approval of purchase requisitions and blanking out the quantity ordered on the receiving department copy of the purchase order.

For the most effective internal control, monthly bank statements should be received directly from the banks and reviewed by the

A.    Cash disbursements accountant

B.    Cash receipts accountant

C.    Controller

D.    Internal auditor

D.

Internal verifications of cash balances generally should be made monthly. Recorded cash on hand and petty cash balances should be compared with cash counts and recorded bank balances should be reconciled to balances shown on bank statements. These verifications should be made by personnel who are not otherwise involved in executing or recording cash transactions to maintain a segregation of functions. The cash receipts accountant, the cash disbursements accountant, and the controller should not reconcile the monthly statements as they are involved in the executing or recording of cash transactions.

For which of the following audit tests would an auditor most likely use attribute sampling? A. Selecting AR for confirmation of account balances; B. Inspecting employee time cards for proper approval by supervisors; C. Making an independent estimate of the amount of a LIFO inventory; D. Examining invoices in support of the valuation of fixed asset additions.

B. Attribute sampling is used to test controls. Inspecting employee time cards for proper approval by supervisors is a test of controls. Controls often relate to authorization, validity, completeness, accuracy, appropriate classification, accounting in conformity with GAAP, and proper period. Look for these terms in identifying which option is a test of control. Words such as accountant balance, amount, valuation, presentation, and disclosure are more likely to relate to substantive test.

If an auditor of an issuer examines purchase orders obtained from the issuer to verify the proper authorization of transactions, then the auditor is conducting

A.    A re-performance.

B.    A confirmation.

C.    An observation.

D.    An inspection.

The correct answer is (D).

While conducting an inspection, an auditor examines documents and records used in internal control, such as authorization forms and procedure manuals. Thus, examining purchase orders to verify the proper authorization of transactions is an example of inspection.

(A) is incorrect because, in a re-performance, the auditor applies the control that the client personnel presumably performed earlier.

(B) is incorrect because confirmation is a substantive procedure to verify the existence of transactions and balances. The given instance is of a test of control to verify the proper authorization of transactions.

(C) is incorrect because while conducting an observation, the auditor watches client personnel performing their regular functions to see if they follow the controls that were designed and implemented.

If an auditor performing an integrated audit identifies one or more material weaknesses in a non-issuer's internal control, the auditor should

A.    Expand the procedures of internal control to identify deficiencies less severe than material weaknesses.

B.    Conclude that the financial statements are materially misstated because of the material weakness in internal control.

C.    Disclaim an opinion on internal control.

D.    Express an adverse opinion on the entity's internal control.

The correct answer is (D).

Material weaknesses are severe enough that they always result in an adverse opinion. In terms of severity, the following are the rankings for control deficiencies and audit opinions.

In terms of severity (least to most): Control Deficiency > Significant deficiency > Material Weakness

In terms of severity (least to most): Unqualified > Qualified > Adverse

(A) is incorrect because the auditor is not required to search for deficiencies less severe than material weaknesses.

(B) is incorrect because there is a possibility that the financial statements are fairly stated even though there are material weaknesses in the internal control.

(C) is incorrect because a disclaimer of opinion should only be given in case of a scope limitation and not on identification of material weaknesses.

If during the audit, the auditor concludes that the materiality for the financial statements as a whole (and, if applicable, materiality level or levels for particular classes of transactions, account balances, or disclosures) is inappropriate and thus revises materiality levels, the auditor is required to

A.    Determine whether it is also necessary to revise performance materiality and whether the nature, timing, and extent of the further audit procedures remain appropriate, but only when the materiality for the financial statements taken as a whole is lowered

B.    Determine whether it is also necessary to revise performance materiality, but only when the materiality for the financial statements taken as a whole is lowered

C.    Determine whether it is also necessary to revise performance materiality and whether the nature, timing, and extent of the further audit procedures remain appropriate if the materiality for the financial statements taken as a whole is revised in either direction

D.    Determine whether it is also necessary to revise performance materiality if the materiality for the financial statements taken as a whole is revised in either direction

The correct answer is (A).

Materiality for the financial statements as a whole (and, if applicable, the materiality level or levels for particular classes of transactions, account balances, or disclosures) may need to be revised as a result of a change in circumstances that occurred during the audit (for example, a decision to dispose of a major part of the entity's business), new information, or a change in the auditor's understanding of the entity and its operations as a result of performing further audit procedures.

For example, if, during the audit, it appears as though actual financial results are likely to be substantially different from the anticipated period-end financial results that were used initially to determine materiality for the financial statements as a whole, the auditor may be required to revise materiality.

If the auditor concludes that a lower materiality than that initially determined for the financial statements taken as a whole (and, if applicable, materiality level or levels for particular classes of transactions, account balances, or disclosures) is appropriate, the auditor should determine whether it is necessary to revise performance materiality and whether the nature, timing, and extent of the further audit procedures remain appropriate. This, of course, should be appropriately documented.

In a probability-proportional-to-size sample with a sampling interval of $10,000, an auditor discovered that a selected AR with a recorded amount of $5000 had an audited amount of $4000. If this were the only misstatement discovered by the auditor, the projected misstatement of this sample would be

$2000 $5,000-4,000-1,000 1,000/5,000=20% $10,000*20%=2,000

In addition to evaluating the frequency of deviations in tests of controls, an auditor should also consider certain qualitative aspects of the deviations. The auditor most likely would give broader consideration to the implications of a deviation if it was: A. The only deviation discovered in the sample; B. Identical to a deviation discovered during the prior year’s audit; C. Caused by an employee’s misunderstanding of instructions; D. Initially concealed by a forged document.

D The auditor should consider both the qualitative and the quantitative aspects of deviations in tests of controls. Qualitative aspects might include whether deviations are indicative of an error or fraud. Such an evaluation is important because fraud is intentional, has implications beyond the direct monetary effect, and requires consideration of the implications for other aspects of the audit. Thus, a deviation initially concealed by a forged document is very serious and deserves broader consideration than a deviation of the same dollar amount due to an error. A is incorrect. The fact that a deviation was the only one discovered would have no importance beyond its impact on the computation of the upper deviation rate; B is incorrect. Discovery of a deviation identical to one discovered during the prior year’s audit is not necessarily cause for additional concern; C is incorrect. Employee misunderstanding of instructions is an inherent limitation of internal control and is not necessarily cause for concern.

In addition to risk assessment procedures, when the auditor is obtaining an understanding of the entity and its environment, which of the following is a related activity the auditor should perform?

A.    The auditor should consider whether information obtained from the auditor’s client acceptance or continuance process is relevant to identifying risks of material misstatement.

B.    If the engagement partner has performed other engagements for the entity, the engagement partner should consider whether there is a need to redetermine materiality for the current audit.

C.    When the auditor intends to use information from procedures performed in previous audits, the auditor should confirm that the previous audit’s workpapers include such procedures.

D.    The engagement partner should hold a discussion with all members of the engagement team about the susceptibility of the entity’s financial statements to material misstatement and the application of the applicable financial reporting framework to the entity’s facts and circumstances.

A.

The auditor should consider whether information obtained from the auditor’s client acceptance or continuance process is relevant to identifying risks of material misstatement.

If the engagement partner has per­formed other engagements for the entity, the engagement partner should consider whether information obtained is relevant to identifying risks of material misstatement. The determination of materiality for a previous audit is not carried forward to the current audit.

When the auditor intends to use information obtained from the auditor’s previous experience with the entity and from audit procedures performed in previous audits, the auditor should determine whether changes have occurred since the previous audit that may affect its relevance to the current audit. Confirmation of the inclusion of the procedures in the previous engagement’s documentation is not required.

The engagement partner is only required to include key members of the engagement team. The engagement partner should determine which matters are to be communicated to engagement team members not involved in the discussion.

In an integrated audit of a non-issuer, which of the following is the responsibility of an auditor with regard to testing controls at a company with multiple business units?

A.    Testing controls over only certain specific risks at all business units of the company.

B.    Testing controls over all risks at all business units of the company.

C.    Testing controls over all risks at business units that are material to the company's consolidated financial statements.

D.    Testing controls over specific risks at business units that are material to the company's consolidated financial statements.

The correct answer is (D).

In an integrated audit of a non-issuer, the auditor must test controls over specific risks at business units that are material to the company’s consolidated financial statements.

The audit of internal control over financial reporting should be integrated with the audit of the financial statements. The objectives of both the audits are not identical. However, the auditor must plan and perform the work to achieve the objectives of both audit of Financial Statements and Internal Control over Financial Reporting. In determining the locations or business units at which to perform tests of controls, AU-C 940 requires that the auditor should assess the risk of material misstatement to the financial statements associated with the location or business unit and correlate the amount of attention devoted to the location or business unit with the degree of risk.

The auditor may eliminate from further consideration locations or business units that, individually or when aggregated with others, do not present a reasonable possibility of material misstatement to the entity's consolidated financial statements.

In assessing the competence of the internal audit function, an external auditor most likely would obtain information about the

A.    Quality of the internal auditors' working paper documentation

B.    Organization’s commitment to integrity and ethical values

C.    Influence of management on the scope of the internal auditors' duties

D.    Organizational level to which the internal audit function reports

A.

The quality of the internal audit function's documentation reflects on their competence. The other answers are related to their objectivity.

Editor’s note: The two key words to remember regarding internal auditors as it pertains to the CPA Exam are: objectivity and competence. Competence relates to the core ability to perform the function, and objectivity relates to how impartial the internal auditors are in relation to the organization they work for.

In assessing the objectivity of internal auditors, an external auditor may

A.    Evaluate the quality control program in effect for the internal auditors.

B.    Examine documentary evidence of the work performed by the internal auditors.

C.    Test a sample of the transactions and balances that the internal auditors examined.

D.    Determine the organizational level to which the internal auditors report.

D.

Answer d., when assessing the internal auditors'; objectivity, the external auditor may obtain information about the organizational status of the internal auditors, including the organizational level to which they report.

The other answer alternatives do not directly relate to their objectivity.

In obtaining an understanding of a manufacturing entity's internal control concerning inventory balances, an auditor most likely would

A.    Analyze the liquidity and turnover ratios of the inventory

B.    Perform analytical procedures designed to identify cost variances

C.    Review the entity's descriptions of inventory policies and procedures

D.    Perform test counts of inventory during the entity's physical count

C.

To obtain an understanding of a manufacturer’s internal control concerning inventory balances, an auditor would review the entity’s descriptions of inventory policies and procedures. Analyzing inventory ratios, performing cost variance analytical procedures, and performing inventory test counts are substantive proce­dures. (Editor note: The key word here is “understanding the entity and therefore of the four answer choices, reviewing policies and procedures would satisfy the objective. The other answer choices are testing procedures that would ascertain the risk assessment i.e. support the auditor’s understanding in this area).

In obtaining an understanding of an entity's internal control in a financial statement audit, an auditor is not required to

A.    Determine whether the control procedures have been implemented

B.    Evaluate the design of the internal control policies

C.    Document the understanding of the entity's internal control

D.    Search for significant deficiencies in the operation of the entity's internal control

D.

While the auditor should be alert for significant deficiencies, the auditor is not required to search for them. Obtaining an understanding of internal controls that are relevant to the audit involves evaluating the design of controls and determining whether they have been implemented. The key elements of this understanding should be documented.

In obtaining an understanding of an entity's internal controls that are relevant to the audit an auditor is required to obtain knowledge about the

A.    Design of the controls

B.    Effectiveness of the controls that have been placed in operation

C.    Consistency with which the controls are currently being applied

D.    By whom and what means the controls were applied

A.

When obtaining an understanding of controls that are relevant to the audit, the auditor should evaluate the design of those controls. The auditor should also determine whether they have been implemented by performing procedures in addition to inquiry of the entity's personnel. (Inquiry alone is not sufficient.)

The incorrect answers relate to testing the operating effectiveness of controls rather than obtaining an understanding of them.

In obtaining an understanding of an entity's internal control, the auditor should obtain an understanding of control activities relevant to the audit, which are those judged necessary to understand in order to

A.    Assess whether operational efficiency has been achieved in accordance with management plans

B.    Assess the risks of material misstatement at the assertion level and design further audit procedures responsive to assessed risks

C.    Determine that management cannot override the controls

D.    Determine that controls have not been circumvented by collusion

B.

The auditor should obtain an understanding of control activities relevant to the audit, which are those judged necessary to understand in order to assess the risks of material misstatement at the assertion level and design further audit procedures responsive to assessed risks. An auditor performs tests of controls later in the audit to obtain sufficient appropriate audit evidence about the operating effectiveness of controls as opposed to operating efficiency.

Collusion and management override are inherent limitations of internal control.

In obtaining an understanding of an entity's internal control, an auditor is required to obtain knowledge about the

#Operating effectiveness of policies and proceduresDesign of policies and procedures

No; Yes

Obtaining an understanding of internal control consists of evaluating the design and implementation of controls. This is not the same as testing the operating effectiveness of controls. The auditor only tests the operating effectiveness of controls when: (1) the auditor’s risk assessment includes an expectation of the oper­ating effectiveness of controls or (2) when it is not possible or practicable to reduce detection risk at the relevant assertion level to an acceptably low level with audit evidence obtained from substantive procedures alone.

Editor note: When the auditor is relying on policies and procedures, the auditor must document such reliance by performing a test of controls in this area.

In obtaining written representations from management, materiality limits ordinarily would apply to repre­sentations related to

A.    Amounts concerning related-party transactions

B.    Irregularities involving members of management

C.    The availability of financial records

D.    The completeness of minutes of directors’ meetings

A.

Materiality limits ordinarily would apply to written representations related to amounts concerning related-party transactions. Materiality considerations would not apply to those representations that are not directly related to amounts included in the financial statements such as the availability of financial records or the completeness of minutes of directors’ meetings. Other items of this nature are management’s acknowledgement of its responsibility for the fair presentation of financial statements in conformity with the applicable financial reporting framework and communications from regulatory agencies concerning noncompliance with or deficiencies in financial reporting practices. Nor would materiality apply to irregularities involving members of management because of the possible effects of fraud on other aspects of the audit.

In order to respond to the increased risks that could be present in the initial audit of an entity, an external auditor should consider the assignment of A. Internal audit personnel with appropriate levels of capabilities and competence. B. Accounting personnel with appropriate levels of capabilities and competence. C. Senior management with appropriate levels of capabilities and competence. D. External audit personnel with appropriate levels of capabilities and competence.

D. Due to an increased level of risk that is present in the initial audit of an entity auditor will assign more experienced staff or those with specialized skills i.e. assign external audit personnel with appropriate levels of capabilities and competence. Senior Management, accounting personnel, and internal audit personnel are the client’s internal management and the auditor will not assign them in response to the increased level of risk.

In planning an audit, an auditor should document in the working papers the auditor's risk assessment of a material misstatement of the financial statements due to fraud. Which of the following should be included in workpaper documentation if risk factors are identified as being present? A. A copy of the report of the risk factor to the company's legal counsel. B. Discussion of the risk factor with the client. C. Investigation of the risk factor. D. Those risk factors identified.

D. Fraud risk factors are events/conditions that indicate fraud. Determination of whether a fraud risk factor is present and whether it is to be considered in assessing fraud risk requires the exercise of professional judgment by the auditor. In planning an audit, an auditor should document in the working papers the auditor's risk assessment of a material misstatement of the financial statements due to fraud. The risk factors if identified must be detailed in the working papers. Answer B is incorrect. The auditor may choose to discuss identified fraud risk factors with the client. However, this is not a required discussion. (Note: If the auditor has identified fraud that occurred or has obtained information that indicates that fraud may exist, then this should be communicated to the appropriate level of MGMT and documentation of this communication should be included in the audit work papers.)

In planning an audit, the auditor’s knowledge about the design of relevant internal control activities should be used to

A.    Identify the types of potential misstatements that could occur.

B.    Assess the operational efficiency of internal control.

C.    Determine whether controls have been circumvented by collusion.

D.    Document the assessed level of control risk.

The correct answer is (A)

The first step in planning an audit for an auditor is to gain an understanding of the design of the relevant control activities. This helps an auditor get a basic understanding of the entity's internal control structure with which he can identify the potential areas with misstatements that could occur.

In planning the audit, the auditor makes judgments about the size of misstatements that will be considered material. It is least likely that these judgments provide a basis for

A.    Determining the nature and extent of risk assessment procedures

B.    Identifying and assessing the risk of material misstatement

C.    Determining whether to accept an engagement and the related fees

D.    Determining the nature, timing, and extent of further audit procedures

C.

Judgments about materiality provide a basis for all of the answer alternatives except determining whether to accept an engagement and the related fees.

In statistical sampling methods used in substantive testing, an auditor most likely would stratify a population into meaningful groups if: A. Probability-proportional-to-size (PPS) sampling is used; B. The population has highly variable recorded amounts; C. The auditor’s estimated tolerable misstatement is extremely small; D. The standard deviation of recorded amounts is relatively small.

B is correct. The auditor may be able to reduce the required sample size by separating items subject to sampling into relatively homogenous groups on the basis of some characteristic related to the specific audit objective A is incorrect. While PPS sampling results in a stratified sample, it is a result of the sampling method employed and does not require the auditor to perform stratification since it occurs automatically; C is incorrect. The estimated tolerable misstatement does not affect the decline to stratify; D is incorrect. The standard deviation of the recorded amounts represents the population’s variability. Therefore, the auditor would be most likely to stratify when the standard deviation is high, not low.

In the audit report on a public company’s financial statements, the principal auditor decides not to make reference to another auditor who audited a client's subsidiary. The principal auditor could justify this decision if, among other requirements, the principal auditor

A.    Issues an unqualified opinion on the consolidated financial statements

B.    Learns that the other auditor issued an unqualified opinion on the subsidiary's financial statements

C.    Is unable to review the audit programs and working papers of the other auditor

D.    Performs procedures to be satisfied as to the audit done by the other auditor

D.

If the principal auditor is satisfied as to the independence and professional reputation of the other auditor and the audit performed by the other auditor, the auditor may be able to express an opinion on the financial statements taken as a whole without making reference. In this case, the principal auditor should not state in the audit report that part of the audit was performed by another auditor because to do so may cause a reader to misinterpret the degree of responsibility being assumed. (Editor note: Whether or not the principal auditor decides to make reference, the principal auditor should make inquiries concerning the professional reputation and independence of the other auditor. The principal auditor also should adopt appropriate measures to assure the coordination of activities in order to achieve a proper review of matters affecting the consolidating or combining of accounts in the financial statements.) Issuing an unqualified opinion is not a justification for the decision not to make reference. Whether or not the other auditor issued an unqualified opinion does not automatically determine whether the principal auditor should make reference. (Editor note: If the report of the other auditor is other than a standard report, the principal auditor should decide whether the reason for the departure is of such nature and significance in relation to the financial statements on which the principal auditor is reporting that it would require recognition in the principal auditor's report.) In some situations, it may be impracticable for the principal auditor to review the other auditor's work or to use other procedures to be satisfied as to the audit performed by the other auditor. In this case, the principal auditor would need to make reference by clearly indicating, in the introductory, scope, and opinion paragraphs, the division of responsibility between the principal auditor and the other auditor in expressing the opinion on the financial statements. (Editor note: The audit report should also disclose the magnitude of the portion of the financial statements audited by the other auditor. This may be done by stating the dollar amounts or percentages of one or more of the following: total assets, total revenues, or other appropriate criteria, whichever most clearly reveals the portion of the financial statements audited by the other auditor. The other auditor may be named but only with express permission and provided the other audit report is presented together with that of the principal auditor.)

Editor note: Regardless of the principal auditor's decision whether to make reference, the other auditor remains responsible for the performance of his or her own work and report. And reference in the report of the principal auditor to the fact that part of the audit was made by another auditor is not to be construed as a qualification of the opinion but rather as an indication of the divided responsibility between the auditors who conducted the audits of various components of the overall financial statements.

In the context of the entity's applicable legal and regulatory framework,

A.    Due to the inherent limitations of an audit, an audit performed in accordance with US GAAS provides only limited assurance that all noncompliance with laws and regulations will be detected or that any contingent liabilities that result will be disclosed

B.    Due to the inherent limitations of an audit, an audit performed in accordance with US GAAS provides no assurance that all noncompliance with laws and regulations will be detected or that any contin¬gent liabilities that result will be disclosed

C.    Due to the inherent limitations of an audit an audit performed in accordance with US GAAS provides only reasonable assurance that all noncompliance with laws and regulations will be detected or that any contingent liabilities that result will be disclosed

D.    Ordinarily, the further removed noncompliance is from the events and transactions reflected in the financial statements, the more likely the auditor is to become aware of, or recognize, the noncompliance.

B.

Due to the inherent limitations of an audit, an audit performed in accordance with US GAAS provides no assurance that all noncompliance with laws and regulations will be detected or that any contingent liabilities that result will be disclosed. Ordinarily, the further removed noncompliance is from the events and transactions reflected in the financial statements, the less (not more) likely the auditor is to become aware of, or recognize, the noncompliance.

Independent internal verification of inventory occurs when employees who

A.    Issue raw materials, obtain material requisitions for each issue and prepare daily totals of materials issued

B.    Compare records of goods on hand with physical quantities, do not maintain the records or have custody of the inventory

C.    Are independent of issuing production orders, update records from completed job cost sheets and production cost reports on a timely basis

D.    Obtain receipts for the transfer of completed work to finished goods, prepare a completed production report

B.

Incompatible functions are those that place any person in a position to both perpetrate and conceal errors or fraud in the normal course of their duties. A well-designed plan of organization separates the duties of authorization, record keeping, and custody of assets. Answers a. and d. do not separate custody and record keeping. Answer c. does not provide verification of inventory.

Inherent limitations of internal control includes

collusion, human error, and management override.

Inherent risk and control risk differ from detection risk in that inherent risk and control risk are

Functions of the client and its environment, while detection risk is not. Inherent risk and control risk differ from detection risk in that they exist independent of the audit; that is, the levels of inherent and control risk are functions of the client audit environments. The auditor has little control over these risks. The auditor can control detection risk through the scope (nature, timing and extent) of the audit procedures formed. Thus, detection risk has an inverse relationship with inherent and control risk. Inherent risk is the susceptibility of an assertion to a material misstatement, assuming there are no related controls. Control risk is the risk that a material misstatement that could occur in an assertion will not be prevented, or detected and corrected, on a timely basis by the entity's internal control. Detection risk is the risk that the auditor will not detect a material misstatement that exists in an assertion. Thus, inherent risk and control risk are functions of the client and its environment while detection risk is not. Inherent risk, control risk, and detection risk are all a part of audit risk. Inherent risk and control risk differ from detection risk in that they exist independently of the audit of financial statements, whereas detection risk relates to the auditor's proce­dures and can be changed at the auditor's discretion. All of the elements of audit risk should be considered in relation to individual account balances, classes of transactions and disclosures; and at the overall financial statement level.

Lapping occurs when an employee withholds funds received by a customer for personal use and fails to apply these receipts of cash or checks to the customer's receivable balance. One of the best methods to guard against lapping is use of a "lock box" system. In this system, customers send their payments directly to the bank, which prevents company employees from having access to payments received. A statement is then sent by the bank to the company so the cash can be applied in the general ledger to the outstanding receivable balance.

Kiting occurs when a check drawn on one bank is deposited in another bank and no record is made of the disbursement in the balance or the first bank until after year-end. Kiting may be used to cover a cash shortage or to pad a company's cash position. Kiting results in an intentional overstatement of cash in the F/S as the cash is simultaneously reflected in 2 different bank accounts. To detect kiting effectively, a bank transfer schedule should be prepared. For any bank-to-bank transfers that occur near year-end, the disbursement date on the check and in the ledger for the disbursing account should precede the receipt date noted by the bank and in the ledger for the receiving account. To ensure that kiting has not occurred, evidence should exist that all deposits in transit and outstanding checks listed on the bank reconciliation at year-end cleared in the next period. This information can be confirmed by using a bank cut off statement.

Management is in a unique position to perpetrate fraud because of management's ability to manipulate accounting records and prepare fraudulent financial statements by overriding controls that otherwise appear to be operating effectively. Management override A. Is presumed to be present in all entities B. Should be considered by the auditor to determine if it is present C. Is considered a fraud risk due to its predictability D. Risk responses should include a review of journal entries near year-end as opposed to throughout the reporting period

A. Although the level of risk of management override of controls (override risk) will vary from entity to entity, the risk is, nevertheless, present in all entities. (Thus, answer A. is correct and answer B. is not.) Even if specific risks of material misstatement (RMM) due to fraud are not identified by the auditor, a possibility exists that management override could occur. Accordingly, the auditor should address this risk apart from any conclu­sions regarding the existence of more specifically identifiable risks by designing and performing audit proce­dures as discussed in the editor note below. Regarding incorrect answer C., due to the unpredictable way in which override could occur, it is a RMM due to fraud and thus a significant risk. Regarding incorrect answer D., one of the procedures required by US GAAS as a response to override risk is to select journal entries (and other adjustments) made at the end of a reporting period and to (not as opposed to) consider the need to test both throughout the period.

Management's emphasis on meeting projected profit goals most likely would significantly influence an entity's control environment when

A.    Internal auditors have direct access to the entity's board of directors.

B.    A significant portion of management compensation is represented by stock options.

C.    External policies established by parties outside the entity affect accounting policies.

D.    The audit committee is active in overseeing the entity's financial reporting policies.

B.

Management’s emphasis on meeting projected profit goals most likely would significantly influence the control environment when a significant portion of management compensation is represented by stock options because it provides an incentive to commit fraud.

Answers A., C., and D. would mitigate this risk factor.

Manual controls would most likely be more suitable than automated controls for which of the following?

A.    Large, unusual, or non-recurring transactions.

B.    High-volume transactions that require additional calculations.

C.    Situations with routine errors that can be predicted and corrected.

D.    Circumstances that require a high degree of accuracy.

A.

The correct answer is (A).

Human involvement is more in case of manual controls. Manual controls are suitable for large, unusual, or non-recurring transactions so that personal attention can be given to each such transaction. As these transactions are of high value (e.g. purchase of a high-value equipment), proper authorization and justification is required for entering into these transactions, which is best served by manual rather than automated controls.

(B), (C) and (D) are incorrect because high volume transactions, situations with routine predictable errors and circumstances requiring high accuracy are best served by automated controls. IT systems could process such transactions faster and more accurately than manual systems.

Memorize the following hierarchy of audit evidence (from most reliable to least reliable)

A. Auditor’s direct personal knowledge & observation; E. External evidence; I. Internal evidence; O. Oral evidence.

Miller Retailing, Inc. maintains a staff of three full-time internal auditors who report directly to the controller. In planning to use the work of internal auditors in obtaining audit evidence, the external auditor most likely will

A.    Place limited reliance on the work of the internal auditors in obtaining audit evidence

B.    Increase the number of procedures performed to test the adequacy of their work

C.    Evaluate whether their level of competence compensates for the lack of support of their objectivity by their organizational status

D.    Avoid using the work of the internal auditors in obtaining audit evidence

D.

The external auditor would most likely avoid using the work performed by the internal auditors. The external auditor should not use the work of the internal audit function in obtaining audit evidence if the external auditor determines that the function's organizational status and relevant policies and procedures do not adequately support the objectivity of internal auditors [or the function lacks sufficient competence; or the function does not apply a systematic and disciplined approach, including quality control.]

In this case, the external auditor would most likely avoid using their work instead of only limiting reliance on it because they report to the controller and, thus, may be reluctant to report weaknesses in the controller’s activities. When assessing the internal auditor's objectivity, the external auditor should consider whether the organizational status of the internal audit function, including the function's authority and accountability, supports the ability of the function to be free from bias, conflict of interest, or undue influence of others to override professional judgments (for example, whether the internal audit function reports to those charged with governance or an officer with appropriate authority, or if the function reports to management, whether it has direct access to those charged with governance). The question does not state that their reporting to management [the controller] is mitigated by direct access to those charged with governance.

Generally, procedures designed to evaluate of the adequacy of their work will provide more information about their competence rather than their objectivity.

Although objectivity and competence may be viewed as a continuum (the more their objectivity is supported by the entity and higher their level of competence, the more likely the external auditor may make use of their work and make use of it in more areas) a significant insufficiency in one, as is the case in this question, cannot be overcome by strength in the other. Additionally, neither a high level of competence nor strong support for their objectivity can compensate for the lack of a systematic and disciplined approach.

Miller Retailing, Inc., maintains a staff of three full-time internal auditors who report directly to the audit committee. In planning to use the internal auditors to help in performing the audit, the independent auditor most likely will

Place limited reliance on the work performed by the internal auditors. 

In assessing the objectivity of internal auditors,the external auditor would be most likely to consider the

A.    Internal auditing standards developed by The Institute of Internal Auditors

B.    Tests of internal control activities that could detect errors and fraud

C.    Materiality of the accounts recently inspected by the internal auditors

D.    Results of the tests of transactions recently performed by the internal auditors

A.

Considering professional standards and whether internal auditors meet them provides an indication regarding their objectivity.The other answer choices relate to the actual audit work and hence would steer more towards the competency side of the internal auditor assessment.

Obtaining an understanding of an internal control involves evaluating the design of the control and determining whether the control has been

A.    Authorized

B.    Implemented

C.    Tested

D.    Monitored

B.

Obtaining an understanding of an internal control involves evaluating the design of the control and determining whether the control has been implemented.

On receiving a client's bank cut off statement, an auditor most likely would trace: A. Prior-year checks listed in the cutoff statement to the year-end outstanding checklist. B. Deposits in transit listed in the cut off statement to the year-end bank reconciliation; C. Checks dated after year-end listed in the cutoff statement to the year-end outstanding checklist; D. Deposits recorded in the cash receipts journal after year-end to the cutoff statement.

A is correct. The auditor should obtain bank cutoff statements that include transactions for 10 to 15 days after year-end. The outstanding checks and deposits in transit at year-end on the bank reconciliation should agree with the information in the bank cutoff statement. B is incorrect. The deposits in transit are listed in the year-end bank reconciliation and traced to actual deposits appearing on the bank cutoff statement. The cutoff statement includes actual deposits received, not deposits in transit; C is incorrect. Checks dated after year-end would not be included in the year-end outstanding checklist; D is incorrect. Deposits recorded in the cash receipts journal after year-end do not effect the cash balance at year-end, and therefore the auditor would not perform audit procedures with respect to those deposits.

Positive confirmation=must respond

Negative confirmation=No news is good news

Prior to evaluating the effect of uncorrected misstatements, the auditor should

A.    Reassess materiality to confirm whether it remains appropriate in the context of the entity’s projected financial results

B.    Reassess materiality to confirm whether it remains appropriate in the context of the entity’s actual financial results

C.    Reassess materiality to confirm whether it remains appropriate in the context of the entity’s latest available interim financial results

D.    Reassess materiality to confirm whether it remains appropriate in the context of the entity’s actual financial results adjusted for uncorrected misstatements

B.

The auditor’s determination of materiality is often is based on estimates of the entity’s financial results because the actual financial results may not yet be known. Therefore, prior to the auditor’s evaluation of the effect of uncorrected misstatements, it may be necessary to revise materiality based on the actual financial results. That said, any significant revision is likely to have been made (based on information the auditor became aware of during the audit) before the auditor evaluates the effect of uncorrected misstatements.

Regarding incorrect answers A. and C., the reassessment should be based on the actual, not the projected or interim, financial results.

Regarding incorrect answer D., the auditor would not adjust the financial statements for the uncor­rected misstatements. The auditor’s objective is to reassess materiality based on the actual financial statements without the corrections.

Prior to the auditor's identification and assessment of the risks of material misstatement, the auditor should consider all of the following except A. The determination of materiality B. The involvement of specialists C. A general understanding of the legal and regulatory framework applicable to the entity and how the entity is complying with that framework D. Results of analytical procedures

D. Which analytical procedures to be applied as risk assessment procedures, not the results of analytical procedures, should be considered prior to the identification and assessment of the risk of material misstatement (RMM). Prior to the auditor's identification and assessment of the RMM, planning includes the need to also consider, of course, the performance of risk assessment procedures other than analytical procedures.

Prior to using internal auditors to provide direct assistance, the external auditor should perform all of the following procedures except

A.    Evaluate the existence and significance of threats to the objectivity of the internal auditors, as well as any safeguards applied to reduce or eliminate the threats

B.    Obtain written acknowledgment from the entity that the internal auditors will be allowed to follow the external auditor’s instructions

C.    Obtain written acknowledgment from the entity that the internal auditors will complete the work assigned to them by the external auditor by the agreed upon dates

D.    Obtain written acknowledgment from the entity that the entity will not intervene in the work the internal auditors perform for the external auditor

C.

The external auditor is not required to obtain written acknowledgment from the entity that the internal auditors will complete the work assigned to them by the external auditor by the agreed upon dates.

Proper segregation of duties reduces the opportunities to allow persons to be in positions to both

A.    Journalize entries and prepare financial statements

B.    Record cash receipts and cash disbursements

C.    Establish internal controls and authorize transactions

D.    Perpetrate and conceal errors and fraud

D.

Assigning different people the responsibilities of authorizing transactions, recording transactions, and maintaining custody of assets is intended to reduce the opportunities to allow any person to be in a position to both perpetrate and conceal errors or fraud in the normal course of their duties.

Incorrect answers A. and B. list record keeping tasks. Incorrect answer C. lists tasks that involve authorization.

Regarding fraud, the auditor is primarily concerned with A. Fraudulent financial reporting B. Fraud resulting from misappropriation of assets C. Making a legal determination of whether fraud has actually occurred D. Fraud that causes a material misstatement in the financial statements

D. The auditor is primarily concerned with fraud that causes a material misstatement in the financial statements. Two types of intentional (fraudulent) misstatements are relevant to the auditor—misstatements resulting from fraudulent financial reporting and misstatements resulting from misappropriation of assets. Obviously, the auditor is concerned with both answers A. and B., but neither, by itself, is the best answer to the question. Although the auditor may suspect or, in rare cases, identify the occurrence of fraud, the auditor does not make legal determinations of whether fraud has actually occurred.

Regarding identified or suspected noncompliance with laws and regulations, the auditor should include in the audit documentation all of the following except

A.    A description of the identified or suspected noncompliance

B.    A copy of the applicable law or regulation or portion thereof violated when noncompliance is identified

C.    The results of the discussion with management and, when applicable, those charged with governance about any identified or suspected noncompliance

D.    The results of the discussion with other parties inside or outside the entity about any identified or suspected noncompliance

B.

US GAAS does not require the inclusion of a copy of the law or regulation related to the noncom­pliance. Examples of documentation of findings may include copies of records or documents and minutes of discussions held.

Regarding materiality considerations

A.    It is not practicable to design audit procedures to detect misstatements that could be material solely because of their nature; thus, the auditor does not make qualitative considerations.

B.    Auditors are aware that most users of financial statements have limited backgrounds in accounting.

C.    The materiality determined when planning the audit does not necessarily establish an amount below which uncorrected misstatements, individually or in the aggregate, will always be evaluated as immaterial.

D.    Risk and materiality are mainly considered during the planning phase of an audit.

C.

The materiality determined when planning the audit does not necessarily establish an amount below which uncorrected misstatements, individually or in the aggregate, will always be evaluated as immaterial. The circumstances related to some misstatements may cause the auditor to evaluate them as material even if they are below materiality.

Although it is not practicable to design audit procedures to detect misstatements that could be material solely because of their nature (that is, qualitative considerations), the auditor considers not only the size but also the nature of uncorrected misstatements, and the particular circumstances of their occurrence, when evaluating their effect on the financial statements.

One of the assumptions auditors make about financial statement users is that they have a reasonable knowledge of business and economic activities and accounting and a willingness to study the information in the financial statements with reasonable diligence.

Materiality and audit risk are considered throughout the audit.

Regarding materiality,

A.    When establishing the overall audit strategy, the auditor should determine materiality for each relevant class of transactions, account balance, or disclosure.

B.    When determining materiality, one factor that may affect the identification of an appropriate benchmark is its relative volatility.

C.    Auditors are aware that most financial statement users are more likely to rely on financial advisors to help them make decisions on the basis of the information in the financial statements rather than studying the information themselves.

D.    Exercising professional judgment, the auditor should set materiality based on a percentage applied to a benchmark appropriate to the entity.

B.

When determining materiality, one factor that may affect the identification of an appropriate benchmark is its relative volatility.

When establishing the overall audit strategy, the auditor should determine materiality for the financial statements as a whole. Only if, in the specific circumstances of the entity, one or more particular classes of transactions, account balances, or disclosures exist for which misstatements of lesser amounts than materiality for the financial statements as a whole could reasonably be expected to influence the economic decisions of users, then, taken on the basis of the financial statements, the auditor also should determine the materiality level or levels to be applied to those particular classes of transactions, account balances, or disclosures.

One of the assumptions auditors make about financial statement users is that users make reasonable economic decisions on the basis of the information in the financial statements. US GAAS does not assume that is it is more likely that a financial advisor will be consulted.

Determining materiality does involve the exercise of professional judgment. A percentage is often applied to a chosen benchmark as a starting point in determining materiality for the financial statements as a whole; however, this methodology is not required.

Regarding risk assessment procedures related to fraud A. Inquiries should be made of management and, when appropriate, those charged with governance. B. Analytical procedures should include procedures related to revenue accounts. C. If fraud risk factors are present, the auditor should rank them in order of importance and investigate accordingly. D. Due to the sensitivity of the subject matter, auditors should only inquire of management and, when appropriate, those charged with governance whether they have knowledge of any actual, suspected, or alleged fraud.

Analytical procedures related to revenue accounts are required to be performed with the objective of identifying unusual or unexpected relationships that may indicate a material misstatement due to fraudulent financial reporting. (Editor Note: These procedures should be also performed through the end of the reporting period when forming an overall audit conclusion and to evaluate whether the accumulated results of all auditing procedures affect the assessment of the risks of material misstatement (RMM) due to fraud made earlier in the audit or indicate a previously unrecognized RMM due to fraud.) Regarding incorrect answer A., inquiries should be made of both management and those charged with governance. The only circumstance allowed by US GAAS that does not require specific inquiries (regarding oversight of management’s processes for identifying and responding to the risks of fraud and the related controls) to be made of those charged with governance is when they are all involved in managing the entity and thus no oversight exists separate from management. Regarding incorrect answer C., fraud risk factors cannot easily be ranked in order of importance and may not necessarily indicate the existence of fraud; however, they have often been present in circumstances in which fraud has occurred and, therefore, may indicate risks of material misstatement (RMM) due to fraud. Their significance varies widely. Accordingly, the determination of whether a fraud risk factor is present and whether it is to be considered in assessing the RMM due to fraud requires the exercise of professional judgment. That said, the auditor is required to evaluate whether the information obtained from the risk assessment procedures and related activities performed indicates that one or more fraud risk factors are present. Regarding incorrect answer D., both management and those charged with governance; and others within the entity as appropriate should be asked whether they have knowledge of any actual, suspected, or alleged fraud.

Regarding the auditor's consideration of compliance with laws and regulations, the auditor

A.    Is required to obtain a general understanding of the legal and regulatory framework applicable to the entity and the industry in which the entity operates and how the entity is complying with that framework

B.    Is required to inspect correspondence, if any, with the licensing or regulatory authorities that enforce laws and regulations generally recognized to have a direct effect on the determination of material amounts and disclosures in the financial statements

C.    Is required to consult with the entity’s external legal counsel

D.    Is required, even when no instances of noncompliance are identified or suspected, to presume that the risk of noncompliance is at least moderate

A.

As part of obtaining an understanding of the entity and its environment, the auditor should obtain a general understanding of the legal and regulatory framework applicable to the entity and the industry or sector in which the entity operates and how the entity is complying with that framework.

Incorrect answer B. is not a requirement for “direct effect” (on the determination of financial statement amounts and disclosures) laws and regulations. The audit procedure to inspect correspondence, if any, with relevant licensing or regulatory author­ities is required to help identify instances of noncompliance with other laws and regulations that may have a material effect on the financial statements. The auditor is also required to make inquiries of management and, when appropriate, those charged with governance about whether the entity is in compliance with such laws and regulations.

Editor Note:  Although neither of these specific procedures just described are required for “direct effect” laws and regulations, they may be performed—the auditor should obtain sufficient appropriate audit evi­dence regarding material amounts and disclosures in the financial statements that are determined by the provisions of those laws and regulations generally recognized to have a direct effect on their determination.

Regarding incorrect answer C., when the auditor identifies or suspects noncompliance, the auditor may consider, but is not required, to consult with the entity’s external legal counsel. Under these circumstances, the auditor should discuss the matter with management and, when appropriate, those charged with governance. If they do not provide sufficient information to the auditor that the entity is in fact in compliance with laws and regulations, the auditor may consider it appropriate to consult with the entity’s in-house legal counsel or external legal coun­sel about the application of the laws and regulations to the circumstances, including the possibility of fraud, and the possible effects on the financial statements. The auditor may request management to arrange for such consultation with the entity’s legal counsel. If it is not considered appropriate to consult with the entity’s legal counsel or if the auditor is not satisfied with the legal counsel’s opinion, the auditor may consider it appropriate to consult the auditor’s own legal counsel.

Regarding incorrect answer D., the auditor is not required to presume a level of risk associated with noncompliance. In the absence of identified or suspected noncompliance, the auditor is not required to perform audit procedures regarding the entity’s compliance with laws and regulations, except for those covered in this answer explanation and requesting written representations from management regarding the entity’s compliance with laws and regulations. And, of course, during the audit, the auditor should remain alert to the possibility that other audit procedures applied may bring instances of noncompliance or suspected noncompliance to the auditor’s attention.

Regarding the auditor's reference to the auditor's external specialist in the audit report,

A.    The auditor should not refer to the work of the specialist in an audit report containing an unmodified opinion

B.    The auditor makes reference to the work of the specialist when the work of that specialist relates to the auditor’s conclusions regarding a significant risk

C.    The auditor makes reference to the work of the specialist if the terms of the agreement with the specialist stipulate that acknowledgment will be given in the audit report to the specialist

D.    If the auditor makes reference to the work of the specialist, the auditor should obtain the specialist’s permission

Answer a., the auditor should not refer to the work of the specialist in an audit report containing an unmodified opinion. Regarding incorrect answer b., the auditor makes reference to the work of an auditor's external specialist in the audit report because such reference is relevant to an understanding of a modification to the auditor's opinion; not when the work of the specialist relates to the auditor's conclusions regarding a significant risk. Regarding incorrect answer c., the auditor should not agree to terms that stipulate the acknowledgment of the specialist in the audit report.

Regarding incorrect answer d., it may be appropriate to refer to the auditor's external specialist in an audit report containing a modified opinion to explain the nature of the modification. In such circumstances, the auditor may need the permission of the auditor's specialist before making such a reference, but it is not a requirement per US GAAS.

Regarding the planning of an audit engagement,

A.    The auditor’s primary objective in planning is to provide a common understanding of the terms of the engagement.

B.    The detailed audit plan provides a basis for the overall audit strategy.

C.    Planning for further audit procedures should be completed before beginning the performance of any further audit procedures to ensure an effective audit.

D.    Planning the nature, extent, and timing of specific further audit procedures depends on the outcome of risk assessment procedures.

D.

Planning the nature, extent, and timing of specific further audit procedures depends on the outcome of risk assessment procedures, which is why planning risk assessment procedures occurs early in the audit pro­cess.

Confirming that a common understanding of the terms of the audit engagement exists between the auditor and the client—a preliminary engagement activity—is an objective of the appropriate acceptance of an engagement; not the objective of planning the audit. The primary objective of the auditor is to plan the audit so that it will be performed in an effective manner.

The overall audit strategy provides a basis for the detailed audit plan; not vice versa. Once the overall audit strategy has been established, an audit plan can be developed to address the various matters identified in the overall audit strategy, taking into account the need to achieve the audit objectives through the efficient use of the auditor’s resources. That said, the establishment of the overall audit strategy and the detailed audit plan are not necessarily discrete or sequential processes but are closely interrelated because changes in one may result in consequential changes to the other. (The audit plan is more detailed than the overall audit strategy in that it includes the nature, timing, and extent of audit procedures to be performed by engagement team members.)

Planning for audit procedures takes place over the course of the audit as the audit plan for the engagement develops. The auditor may begin the execution of further audit procedures for some classes of transactions, account balances, and disclosures before planning all remaining further audit procedures.

Regarding the testing of controls:

A.    If audit evidence is obtained during an interim period, the auditor is not required to obtain additional evidence for the remaining period unless there have been significant changes.

B.    The auditor should obtain more persuasive audit evidence the greater the reliance the auditor places on the effectiveness of a control.

C.    If deviations from controls upon which the auditor intended to rely are detected, the auditor should perform substantive procedures instead.

D.    The auditor should evaluate the operating effectiveness of controls based on tests of controls without consideration of misstatements detected by substantive procedures.

B.

In designing and performing tests of controls, the auditor should obtain more persuasive audit evidence the greater the reliance the auditor places on the effectiveness of a control. Regarding incorrect answer a., the auditor is required to determine the additional audit evidence to be obtained for the remaining period. Of course, if there have been significant changes, the auditor's response will be different than when no changes have occurred; however, additional evidence would need to be obtained to determine there were no changes. Regarding incorrect answer c., if deviations from controls upon which the auditor intends to rely are detected, the auditor should make specific inquiries to understand these matters and their potential consequences (not just simply switch to performing substantive procedures). The auditor should also determine if the tests on controls already performed will suffice. If not, the auditor should determine if either additional tests of controls or substantive procedures are needed to address the potential risks of misstatement.

Regarding incorrect answer d., when evaluating the operating effectiveness of relevant controls, the auditor should evaluate whether misstatements that have been detected by substantive procedures indicate that controls are not operating effectively. However, the absence of misstatements detected by substantive procedures does not provide audit evidence that controls related to the relevant assertion being tested are effective.

Required preliminary engagement activities include all of the following except

A.    Performing quality control procedures related to the continuance of the client relationship and the specific audit engagement

B.    Evaluating compliance with relevant ethical requirements

C.    Establishing an understanding of the terms of the engagement

D.    Consideration of factors that are significant in directing the engagement team’s efforts

D.

The auditor should consider the factors that, in the auditor's professional judgment, are significant in directing the engagement team's efforts when establishing the overall audit strategy, rather than during the earlier preliminary stage.

The other answer alternatives comprise all of the required preliminary engagement activities. Performing them at the beginning of the current audit engagement assists the auditor in identifying and evaluating events or circumstances that may adversely affect the auditor’s ability to plan and perform the audit engagement. Their performance should enable the auditor to plan an engagement for which the auditor maintains the necessary independence and ability to perform the engagement; has no issues with management integrity; or has no misunderstanding with the entity about the terms of the engagement.

Sample deviation rate+Allowance for sampling risk=Upper deviation rate

The allowance for sampling risk recognizes that it is likely that what we found in the sample isn't exactly what we would find in the population. Assume a population of 1000 items, a sample of 100 items, and a sample deviation rate of 4% (4 deviations out of 100). If the upper deviation rate (from a table)is 8.5%, this implies a 4.5% allowance for sampling risk. Conversely, should the examiners provide the allowance for sampling risk (say, 3.6%), it would be added to the sample deviation rate (4%) to find an upper deviation rate of 7.6%.

Sample size will increase as the following increase (direct relationship): -Expected misstatement; -Standard deviation (population variability); -Assessed level of risk

Sample size will decrease as the following increase (inverse relationship): -Tolerable misstatement; -Acceptable level of risk.

Sampling risk in substantive testing=Variable sampling 1. Risk of incorrect acceptance=beta risk: is the risk that the sample supports the conclusion that the recorded account balance is not materially misstated when in fact it is materially misstated (i.e., sample results mistakenly indicate a material misstatement. ) This is auditor's concern/fear.

2. Risk of incorrect rejection=alpha risk: is the risk that the sample supports the conclusion that the recorded account balance is materially misstated when in fact it is not materially misstated (i.e., sample results mistakenly indicate a material misstatement)

Sampling risks in test of controls=Attribute sampling 1. Risk of assessing control risk to low=beta risk=risk of over reliance: is the risk that the assessed level of control risk based on the sample is less than the true risk based on the actual operating effectiveness of the control (i.e., sample results indicate a lower deviation偏离 rate than actually exists in the population)

2. Risk of assessing control risk too high=alpha risk=risk of under reliance: is the risk that the assessed level of control risk based on the sample is greater than the true risk based on the actual operating effectiveness of the control (i.e., sample results indicate a greater deviation rate than actually exists in the population)

SEC rules state that independence is impaired if

the accounting firm, covered persons, and his or her immediate family members have a broker-dealer account with an audit client if the account includes any asset other than cash or securities that are covered by the Securities Investor Protection Act.

Services provided by a service organization are relevant to the audit of a user entity’s financial statements when those services and the controls over them affect the user entity's information system, including related business processes, relevant to financial reporting. A user auditor should consider a service organization's services a part of an entity's information system if they affect any of the following except

A.    The classes of transactions in the user entity's operations that are significant to the entity's financial statements

B.    The procedures within both IT and manual systems by which the user entity’s transactions are initiated, authorized, recorded, processed, corrected as necessary, transferred to the general ledger, and reported in the financial statements

C.    The financial reporting process used to prepare the user entity’s financial statements, including significant accounting estimates and disclosures

D.    Transactions that are specifically authorized by the entity, such as the processing of checking account transactions by a bank or the processing of securities transactions by a broker

D.

US GAAS regarding service organizations do not apply to services that are limited to processing an entity’s transactions that are specifically authorized by the entity, such as the processing of checking account transactions by a bank or the processing of securities transactions by a broker. Nor do they apply to the audit of transactions arising from an entity that holds a proprietary financial interest in another entity, such as a partner­ship, corporation, or joint venture, when the partnership, corporation, or joint venture performs no processing on behalf of the entity. In addition to the other answer alternatives, a service organization’s services are part of a user entity’s information system, including related business processes, relevant to financial reporting if these services affect any of the following: (1) the related accounting records, supporting information, and specific accounts in the user entity’s financial statements that are used to initiate, authorize, record, process, and report the user entity’s transactions; this includes the correction of incorrect information and how information is trans­ferred to the general ledger; the records may be in either manual or electronic form; (2) How the user entity’s information system captures events and conditions, other than transactions, that are significant to the financial statements; and (3) controls surrounding journal entries, including nonstandard journal entries used to record nonrecurring, unusual transactions, or adjustments.

Sound internal control procedures dictate that defective merchandise returned by customers should be presented initially to the

A.    Accounts receivable supervisor

B.    Receiving clerk

C.    Shipping department supervisor

D.    Sales clerk

B.

For sound internal controls, all receipts for goods, including returned goods or materials, should be handled by the receiving clerk. Receiving reports should be prepared for all items received. Employees who have recording responsibilities should not also have custody of the related assets.

The acceptable level of detection risk is inversely related to the A. Assurance provided by substantive tests B. Risk of misapplying auditing procedures C. Preliminary judgment about materiality levels D. Risk of failing to discover material misstatements

A. Detection risk relates to substantive audit procedures and is managed by the auditor's response to the risk of material misstatement (RMM). For a given level of audit risk, detection risk should bear an inverse relationship to the RMM at the relevant assertion level. As the RMM increases, the level of detection risk that can be accepted by the auditor decreases (and the need for the assurance provided by substantive tests increases). Likewise, as the RMM decreases, the level of detection risk that can be accepted by the auditor increases. (However, the auditor should perform substantive procedures for all relevant assertions related to material classes of transactions, account balances, and disclosures regardless of the assessed level of risk.) Regarding incorrect answer B., the risk of misapplying audit procedures is a part of detection risk. Regarding incorrect answer C., the preliminary judgment about materiality levels provides a basis for, i.e., is made prior to, the auditor’s identification and assessment of the RMM. Thus, as detection risk is set by the auditor in response to the RMM, the setting of the preliminary materiality levels is unrelated to detection risk. Moreover, the judg­ment about what constitutes a material misstatement is not related to the determination of the degree of risk the auditor is willing to accept that a material misstatement will be undetected. Regarding incorrect answer D., the risk of failing to discover material misstatements during an audit is detection risk.

The auditor is required to obtain an understanding of the components of the nature of entity which include all of the following except

A.    Its operations

B.    The entity's selection and application of accounting policies

C.    Its ownership and governance structures

D.    The way that the entity is structured and how it is financed

B.

The entity's selection and application of accounting policies, including the reasons for changes (the auditor should evaluate whether the entity’s accounting policies are appropriate for its business and consistent with the applicable financial reporting framework and accounting policies used in the relevant industry) is another aspect of the entity the auditor is required to understand—distinct from the components of the nature of the entity.

In addition to the other answer alternatives, the components of the nature of an entity that the auditor is required to understand include the types of investments that the entity is making and plans to make, including investments in entities formed to accomplish specific objectives.

Editor Note: The understanding of the nature of an entity enables the auditor to understand the classes of transactions, account balances, and disclosures to be expected in the financial statements.

The auditor should agree, in writing when appropriate, with the auditor's specialist regarding the nature, extent, and timing of communication between the auditor and the specialist, including the form of any report to be provided by the specialist and all of the following except

A.    The respective roles and responsibilities of the auditor and the auditor’s specialist

B.    The nature, scope, and objectives of the work of the auditor’s specialist

C.    Any relevant technical performance standards or other professional or industry requirements that the auditor’s specialist will follow

D.    The need for the auditor’s specialist to observe confidentiality requirements

C.

It often may be relevant when agreeing on the nature, scope, and objectives of the work of the auditor’s specialist to include discussion of any relevant technical performance standards or other professional or industry requirements that the auditor’s specialist will follow, but it is not required.

Editor’s note: If we’re relying on the auditor’s specialist (i.e. our specialist), then we trust that the specialist has the competency to follow her respective standards and guidelines with respect to the area we’re relying on her report for.

The auditor should include in the audit documentation related to materiality considerations

A.    Materiality for the financial statements as a whole, performance materiality, and, if applicable, the materiality level or levels for particular classes of transactions, account balances, or disclosures

B.    If the materiality level or levels for particular classes of transactions, account balances, or disclosures are not determined, the reason why this was not considered necessary

C.    Significant revisions to materiality as the audit progressed

D.    The factors considered in the determination of all amounts if a methodology other than benchmarking was used

A.

The auditor should include in the audit documentation the following amounts and the factors considered in their determination (1) materiality for the financial statements as a whole; (2) if applicable, the materiality level or levels for particular classes of transactions, account balances, or disclosures; (3) performance materiality; and (4) any revision of 1–3 as the audit progressed.

Documentation of the reason why materiality was not determined for any classes of transactions, account balances, or disclosures is not required.

Any revision to a materiality amount should be documented; not just significant revisions.

The factors considered in their determination should be documented regardless of the methodology used.

The auditor suspects that fictitious employees have been placed on the payroll by the entity's payroll supervisor, who has access to payroll records and to the paychecks.

Observe payroll check distribution on a surprise basis.

The auditor suspects that selected employees of the entity received unauthorized raises from the entity's payroll supervisor, who has access to payroll records.

Vouch data in the payroll register to documented authorized pay rates in the human resources department's files.

The auditor's knowledge about the design of relevant internal control policies and procedures primarily assists the auditor in

A.    Identifying the types of potential misstatements that could occur

B.    Assessing the operational efficiency of the internal control

C.    Determining whether controls have been circumvented by collusion

D.    Documenting the assessed level of control risk

A.

The auditor primarily uses the the knowledge about the design of relevant internal control policies and procedures [gaining an understanding of internal control] to (1) identify types of potential misstatements and factors that affect the risks of material misstatement; and (2) in designing the nature, extent, and timing of further audit procedures.

The concept of materiality is applied by the auditor in all of the following instances except

A.    In planning and performing the audit

B.    In evaluating the effect of identified misstatements on the audit

C.    In evaluating the effect of fraud by senior management

D.    In forming the opinion in the audit report

C.

If the auditor identifies a misstatement, whether material or not, and the auditor has reason to believe that it may be the result of fraud and that management (in particular, senior management) is involved, the auditor should reevaluate the assessment of the risks of material misstatement due to fraud and its resulting effect on the nature, timing, and extent of audit procedures to respond to the assessed risks.

For example, an otherwise insignificant fraud may be significant if it involves senior management. In such circumstances, the reliability of evidence previously obtained may be called into question because there may be doubts about the completeness and truthfulness of representations made and the genuineness of accounting records and docu­mentation. There may also be a possibility of collusion involving employees, management, or third parties. In addition to the alternative answers, the concept of materiality is applied by the auditor in evaluating the effect of uncorrected misstatements, if any, on the financial statements.

The controller of a small utility company has interviewed audit firms proposing to perform the annual audit of their employee benefit plan. According to the guidelines of the Department of Labor (DOL), the selected auditor must be A. The firm that proposes the lowest fee for the work required B. Independent for purposes of examining financial information required to be filed annually with the DOL C. Included on the list of firms approved by the DOL D. Independent of the utility company and not relying on its services

B. According to the guidelines of the Department of Labor (DOL), the selected auditor performing the annual audit of the company's employee benefit plan must be independent for purposes of examining financial information required to be filed annually with the DOL. The selected auditor may be a customer of the utility company, does not have to be on any approved list of firms, and does not have to be the lowest fee bidder for the work required.

The element of the audit planning process most likely to be agreed upon with management before imple­mentation of the audit strategy is the determination of the

A.    Evidence to be gathered to provide a sufficient basis for the auditor's opinion

B.    Procedures to be undertaken to discover litigation, claims, and assessments

C.    Pending legal matters to be included in the inquiry of the client's attorney

D.    Timing of inventory observation procedures to be performed

D.

The element of the audit planning process most likely to be agreed upon with the client before implementation of the audit strategy is the timing of inventory observation procedures to be performed. Evidence to be gathered to provide a sufficient basis for the auditor's opinion is solely a matter of auditor judgment. The procedures to be undertaken to discover litigation, claims, and assessments are determined by the auditor. Pending legal matters to be included in the inquiry of the client's attorney are more likely to be discussed after implementation of the audit strategy.

The entity borrowed funds from a financial institution. Although the transaction was properly recorded, the auditor suspects that the loan created a lien on the entity's real state that is not disclosed in its financial statement.

Confirm the terms of borrowing arrangements with the lender.

The establishment of the overall audit strategy, includes all of the following activities except

A.    Identification of the characteristics of the engagement that define its scope

B.    Determination of the reporting objectives of the engagement in order to plan the timing of the audit and the nature of the communications required

C.    Determination of the nature, timing, and extent of resources necessary to perform the engagement

D.    The nature, timing, and extent of planned further audit procedures at the relevant assertion level

D.

The detailed audit plan, not the overall audit strategy, includes a description of the nature, timing, and extent of planned further audit procedures at the relevant assertion level.

In addition to the other answer alternatives, in establishing the overall audit strategy, the auditor should consider the factors that, in the auditor's professional judgment, are significant in directing the engagement team’s efforts; consider the results of preliminary engagement activities; and, when applicable, consider whether knowledge gained on other engagements performed by the engagement partner for the entity is relevant.

The external auditor should determine whether the work of the internal audit function can be used in obtaining audit evidence by evaluating all of the following except

A.    The amount of judgment involved in performing the planned audit procedures

B.    The extent to which the internal audit function’s organizational status and relevant policies and procedures support the objectivity of the internal auditors

C.    The level of competence of the internal audit function

D.    The application by the internal audit function of a systematic and disciplined approach, including quality control

A.

 The more judgment involved in planning and performing relevant audit procedures or evaluating the audit evidence obtained is a factor in the external auditor's decision to perform more of the audit work directly, i.e., to use the work of the internal audit function less, rather than in the determination of whether to use the internal audit function at all to obtain evidence.

The financial accounting standards (and the organizations that are authorized or recognized to promulgate them) to be used by nonissuers for preparing financial statements in accordance with a general-purpose framework include all of the following except

A.    FASB Accounting Standards Codification™ (ASC), issued by the Financial Accounting Standards Board (FASB)

B.    US Generally Accepted Auditing Standards (GAAS), issued by the American Institute of Certified Public Accountants (AICPA)

C.    International Financial Reporting Standards (IFRS), issued by the International Accounting Standards Board (IASB)

D.    Statements of Federal Financial Accounting Standards (SFFAS), issued by the Federal Accounting Standards Advisory Board (FASAB)

B.

US GAAS, issued by the AICPA are auditing standards; not accounting standards. The financial accounting standards (and the recognized organizations that issue them) used to prepare financial statements in accordance with a general-purpose framework (a financial reporting framework designed to meet the common financial information needs of a wide range of users) are: answer A., FASB Accounting Standards Codification™ (ASC), issued by the Financial Accounting Standards Board (FASB); answer C., International Financial Reporting Standards (IFRS), issued by the International Accounting Standards Board (IASB); answer D., Statements of Federal Financial Accounting Standards (SFFAS), issued by the Federal Accounting Standards Advisory Board (FASAB) for US federal government entities; and Statements of Governmental Accounting Standards (GASB Statements), issued by the Governmental Accounting Standards Board (GASB) for US state and local govern­ment entities (not included as an answer alternative in this question).

The five fundamental principles that the International Ethics Standards Board for Accountants (IESBA) requires professional accountants to comply with per the IESBA Code of Ethics for Professional Accountants are

1.integrity; 2.objectivity; 3.professional competence and due care; 4.confidentiality; 5.professional behavior.

The objective of substantive testing

Detect material misstatement in financial statements.

The objectives of the internal control for a production cycle are to provide assurance that transactions are properly executed and recorded, and that

A.    Production orders are prenumbered and signed by a supervisor.

B.    Custody of work in process and of finished goods is properly maintained.

C.    Independent internal verification of activity reports is established.

D.    Transfers to finished goods are documented by a completed production report and a quality control report.

B.

Controlling the access to assets is an important objective of proper inventory control. The other answer alternatives are procedures for satisfying inventory control objectives—not actual internal control objectives.

The purpose of segregating the duties of hiring personnel and distributing payroll checks is to separate the

A.    Human resources function from the controllership function

B.    Administrative controls from the internal accounting controls

C.    Authorization of transactions from the custody of related assets

D.    Operational responsibility from the record keeping responsibility

C.

Incompatible functions are those that place any person in a position to both perpetrate and conceal errors or fraud in the normal course of their duties. A well-designed organization plan separates authorization, record keeping, and asset custody.

The risk of material misstatement at the assertion level consists of which of the following sets of two components?

Inherent risk and control risk At the assertion level, the risk of material misstatement (RMM) consists of (1) inherent risk, the suscep­tibility of an assertion about a class of transaction, account balance, or disclosure to a misstatement that could be material, either individually or when aggregated with other misstatements, before consideration of any related controls, and (2) control risk, the risk that a misstatement that could occur in an assertion about a class of transaction, account balance, or disclosure and that could be material, either individually or when aggregated with other misstatements, will not be prevented, or detected and corrected, on a timely basis by the entity’s internal control. Editor Note: The RMM is the risk that the financial statements are materially misstated prior to the audit. It exists at both the overall financial statement level and the assertion level. (RMM at the overall financial statement level refers to those risks that relate pervasively to the financial statements as a whole and potentially could affect many assertions.) Regarding the incorrect answers: The RMM is a part of audit risk. Audit risk is the risk that the auditor expresses an inappropriate audit opinion when the financial statements are materially misstated. Audit risk is a function of the risks of material misstatement and detection risk. Detection risk is the risk that the procedures performed by the auditor to reduce audit risk to an acceptably low level will not detect a misstatement that exists and that could be material, either individually or when aggregated with other misstatements.

The risk of not detecting a material misstatement resulting from fraud A. Is higher when employees collude than when management is involved in fraud B. Is higher than the risk of not detecting one resulting from the misappropriation of assets C. Is higher than the risk of not detecting one resulting from error D. Is higher than the risk of not detecting one resulting from error, excluding omissions

C The risk of not detecting a material misstatement resulting from fraud is higher than the risk of not detecting one resulting from error. This is because fraud may involve sophisticated and carefully organized schemes designed to conceal it, such as forgery, deliberate failure to record transactions, or intentional misrep­resentations being made to the auditor. Regarding incorrect answer A., the risk of the auditor not detecting a material misstatement resulting from management fraud is greater than for employee fraud (even when employ­ees collude) because management is frequently in a position to directly or indirectly manipulate accounting records, present fraudulent financial information, or override control procedures designed to prevent similar frauds by other employees. Regarding incorrect answer B., it is nonsensical. Misappropriation (theft) of assets is a type of fraud. Regarding incorrect answer D., the risk of not detecting a material misstatement resulting from fraud is higher than the risk of not detecting one resulting from error whether or not the error involves an omission.

The risk that an auditor will conclude, based on substantive tests, that a material error does not exist in an account balance when, in fact, such error does exist is referred to as A. Sampling risk B. Detection risk C. Nonsampling risk D. Inherent risk

B Detection risk is the risk that audit procedure will incorrectly lead to a conclusion that a material misstatement does not exist in an account balance when in fact such a misstatement does exist. Option (A) is incorrect because sampling risk is a risk of drawing the wrong conclusion from the sample because the sample is not representative of the population. Option (C) is incorrect because non-sampling risk is the risk due to auditor's failure and not because of sample Option (D) is incorrect because inherent risk is the risk that material misstatement of an assertion will take place in the absence of any internal controls. Editor’s note: A tip here is that sampling was not performed, but a substantive test was. Therefore, choices A and C could be eliminated, giving you a 50% chance to correctly answer this question. Furthermore, inherent risk relates to an auditor’s judgment prior to the commencement of testing, so that answer could have been eliminated as well.

The use of which of the following word(s) in the IESBA Code of Ethics for Professional Accountants imposes a requirement? A. Must B. Shall C. Should D. Is required

B. The word shall is used to impose a requirement in the IESBA Code of Ethics for Professional Accountants.

The user auditor should apply the same procedures with respect to the services provided by a subservice organization as a service organization when the user auditor plans to use a type 1 or type 2 report that

A.    Was for an engagement for a subservice organization that did not require complementary controls

B.    Was prepared using the carve-out method

C.    Contains a system description that is as of a date or for a period that precedes the beginning of the audit of the user entity

D.    Was prepared by a service auditor who was not independent of the user entity

B.

If the user auditor plans to use a type 1 or a type 2 report that excludes the services provided by a subservice organization, i.e., was prepared using the carve-out method instead of the inclusive method, and those services are relevant to the audit of the user entity’s financial statements, the same US GAAS require­ments imposed on the user auditor that are applicable to the service organization are applicable to the subser­vice organization. None of the other answers describe situations that require this response by the user auditor.

Regarding incorrect answer A., the absence of complementary controls does not; as such controls may not be applicable.

Regarding incorrect answer C., the user auditor may use a service auditor’s report to support the user auditor’s understanding of the service organization or, in the case of a type 2 report, as audit evidence that controls at the service organization are operating effectively. When a type 1 or 2 report is used to support the user auditor’s understanding and the description of the service organization’s system is as of a date or for a period that precedes the beginning of the period under audit, the user auditor may perform procedures to update the information in a type 1 or type 2 report to remedy this matter.

Regarding incorrect answer D., a service auditor should be independent of the service organization, but the service auditor need not be independent of the user entity.

To what degree, if at all, is a significant deficiency related to a material weakness?

A.    It is less severe than a material weakness.

B.    It is more severe than a material weakness.

C.    It is unrelated to a material weakness.

D.    It is equivalent to a material weakness.

The correct answer is (A).

A significant deficiency is less severe than a material weakness. A significant deficiency is a control deficiency, or a combination of control deficiencies, in internal control over financial reporting that is less severe than a material weakness, yet important enough to examine further.

A material weakness is also a control deficiency, or combination of control deficiencies, in internal control over financial reporting, such that there is a reasonable possibility that a material misstatement of the company's annual or interim financial statements will not be prevented or detected on a timely basis.

Tracing shipping documents to prenumbered sales invoices provides evidence that

A.    No duplicate shipments or billings occurred.

B.    Shipments to customers were properly invoiced.

C.    All goods ordered by customers were shipped.

D.    All prenumbered sales invoices were accounted for.

B.

Tracing from shipping documents to the sales invoice provides evidence that shipments to customers were properly invoiced.

Tracing starts with the source documents and traces forward to provide assurance that the event is being given proper recognition in the books and records. (Expenses and liabilities are not understated) Evidence of completeness

Vouching is directional testing in which the auditor examines support for what has been recorded in the records and statements, going from the F/Ss back to supporting documents (Revenue and assets are not overstated). Evidence of existence.

When an auditor increases the assessed level of control risk because certain control activities were determined to be ineffective, the auditor most likely would increase the A. Level of detection risk B. Extent of tests of details C. Level of inherent risk D. Extent of tests of controls

B. As the assessed level of control risk increases, the acceptable level of detection risk decreases. To decrease detection risk, an auditor changes the nature, extent, or timing of tests of details. To decrease detec­tion risk, an auditor increases the extent of test of details. The level of inherent risk doesn't change, although an auditor may change the assessment of the level of inherent risk. As inherent risk is the susceptibility of an assertion to misstatement, assuming no related internal controls, the assessment of inherent risk level rarely changes due to a change in control risk. Increased testing of controls that the auditor already has determined to be ineffective provides no benefit. Editor’s note: In Layman's terms, because control testing didn’t provide you as the auditor with comfort surrounding an account balance, you must now turn to increase your substantive testing in order to get comfort over an account balance (or conclude on whether there are misstatements).

When an auditor of a parent non-issuer is also the auditor of a component, then each of the following factors would ordinarily influence the decision to obtain a separate engagement letter from the component, except

A.    The legal requirements regarding the appointment of the auditor.

B.    Whether a separate audit report is to be issued on the component.

C.    Whether there has been any turnover of the component's board members.

D.    The degree of independence of the component management from the parent entity.

The correct answer is (C).

Under the Auditing Standards, when the auditor of a parent entity is also the auditor of a component, the factors that may influence the decision whether to obtain a separate audit engagement letter from the component include the following:

Who engages the component auditor.

Whether a separate auditor's report is to be issued on the component.

Legal requirements regarding the appointment of the auditor.

Degree of ownership by the parent.

Degree of independence of the component management from the parent entity.

Thus, any turnover of the component's board members would not ordinarily influence the decision to obtain a separate engagement letter from the component.

When an entity uses a trust company as custodian of its marketable securities, the possibility of concealing fraud most likely would be reduced if the

A.    Trust company has no direct contact with the entity employees responsible for maintaining investment accounting records.

B.    Securities are registered in the name of the trust company, rather than the entity itself.

C.    Interest and dividend checks are mailed directly to an entity employee who is authorized to sell securities.

D.    Trust company places the securities in a bank safe-deposit vault under the custodian's exclusive control.

A.

The fact that only the trust company has access to the securities should prevent unauthorized entity personnel with record keeping responsibility from conspiring or colluding to misappropriate the securities. Trust company employees or management potentially could sell or otherwise mismanage the assets.

Answers C. and D. put custody and record keeping functions in the hands of one party which could result in unauthorized transactions.

When audit procedures relate to account balances, the most relevant assertions are (CVER)

Completeness; Valuation, Allocation, and accuracy; Existence and occurrence; Rights and obligations.

When commenting in a comfort letter on information other than audited financial statements, an auditor should

A.    State that nothing else has come to the auditor’s attention that would be of interest to the requesting party as a result of carrying out the specified procedures

B.    State that the auditor has applied procedures that the auditor determined to be necessary or sufficient for the requesting party’s purposes

C.    Describe the criteria specified by the requesting party

D.    Use universally understood and accepted terms such as reconcile, check, or test

C.

When commenting in a comfort letter on information other than audited financial statements, the auditor should describe the criteria specified by the underwriter or other requesting party. The auditor should not make the statements or use the terms (which are of uncertain meaning; they are not universally understood) described in the other answer alternatives.

When considering internal control, an auditor should be aware of the concept of reasonable assurance, which recognizes that

A.    Internal control policies and procedures may be ineffective due to mistakes in judgment and personal carelessness.

B.    Adequate safeguards over access to assets and records should permit an entity to maintain proper accountability.

C.    Establishing and maintaining internal control is an important responsibility of management.

D.    The cost of an entity's internal control should not exceed the benefits expected to be derived.

A.

Internal control, no matter how effective, can provide an entity with only reasonable, but not absolute, assurance of achieving an entity's financial reporting objectives. The likelihood of their achievement is affected by limitations inherent to internal control. These include the realities that human judgment in decision making can be faulty and that breakdowns in internal control can occur because of human failures such as simple errors or mistakes. While the other answers are generally true statements, they are not part of the concept of reasonable assurance.

When determining whether related-party transactions have been properly accounted for in the financial statements, the auditor should be most concerned that

A.    The transactions represent the appropriate legal form

B.    The method of accounting for the transactions reflects that related parties are involved

C.    The financial statements recognize the substance of the transactions

D.    All related parties have been identified

C.

The auditor should be aware that the substance of a particular transaction could be significantly different from its form and that financial statements should recognize the substance of particular transactions rather than merely their legal form. Established accounting principles ordinarily do not require transactions with related parties to be accounted for on a basis different from that which would be appropriate if the parties were not related generally, the auditor should view related-party transactions within the framework of existing pronouncements. The identification of related parties relates more to the disclosure concern rather than the auditor's review for their proper accounting.

When developing the audit strategy and audit plan for an issuer, the auditor should evaluate whether the following matters are important to the company’s financial statements and internal control over financial reporting and, if so, how they will affect the auditor’s procedures, except for

A.    The relative complexity of the company’s operations

B.    The response to the auditor’s letter of audit inquiry to the client’s lawyer for the current audit

C.    Matters relating to the company’s business, including its organization, operating characteristics, and capital structure

D.    The extent of recent changes, if any, in the company, its operations, or its internal control over finan¬cial reporting

B.

The auditor should evaluate legal or regulatory matters of which the company is aware; however, the response to the auditor’s letter of inquiry to the client’s lawyer would not be available in the planning stage; it should be dated as close as possible to the date of the audit report. Additional matters for evaluation during the planning stage are knowledge of the company’s internal control over financial reporting obtained during other engagements performed by the auditor; matters affecting the industry in which the company operates, such as financial reporting practices, economic conditions, laws and regulations, and technological changes; the auditor’s preliminary judgments about materiality, risk, and, in integrated audits, other factors relating to the determination of material weaknesses; control deficiencies previously communicated to the audit committee or management; the type and extent of available evidence related to the effectiveness of the company’s internal control over financial reporting; preliminary judgments about the effectiveness of internal control over financial reporting; public information about the company relevant to the evaluation of the likelihood of material financial statement misstatements and the effectiveness of the company’s internal control over financial reporting; and knowledge about risks related to the company evaluated as part of the auditor’s client acceptance and retention evaluation.

When developing the group audit plan, the group engagement team is required

A.    To determine the timing of the group engagement team’s interaction with component auditors

B.    To determine whether the audit report on the group financial statements will make reference to the audit of each component auditor

C.    To determine whether the audit report on the group financial statements will make reference to the audit of either all or none of the component auditors

D.    To include the component auditors in a brainstorming session regarding the risks of material misstatement of the group financial statements, including the risks of fraud

Answer b., when developing the group audit plan, the group engagement team is required to determine whether the audit report on the group financial statements will make reference to the audit of a component auditor. Editor Note: The group engagement team should also assess the extent to which the group engagement team will use the work of component auditors. Regarding incorrect answer a., when developing the group audit plan the group engagement team is required to assess the extent to which the group engagement team will use the work of component auditors; not determine the timing of the interaction. A determination of the nature, extent, and timing of its involvement in the work of component auditors [interaction] is required when the auditor of the group financial statements is assuming responsibility for the work of a component auditor and the engagement team is deciding the work to be performed by it or on its behalf by the component auditors on the financial information of the components. Regarding incorrect answer c., in group audits involving two or more component auditors, the decision to make reference to the audit of a component auditor is made individually for each component auditor, regardless of the decision whether to refer to any other component auditor. The auditor of the group financial statements may make reference to any, all, or none of the component auditors.

Regarding incorrect answer d., the key members of the engagement team are required to discuss the susceptibility of an entity to material misstatement of the financial statements due to fraud or error, specifically emphasizing the risks due to fraud. In a group audit, these discussions also may include the component auditors, but their inclusion is not a requirement. The group engagement partner’s determination of who to include in the discussions, how and when they occur, and their extent is affected by factors, such as prior experience with the group.

When is timely, i.e., as soon as practicable, communication of matters involving noncompliance with laws and regulations to those charged with governance required?

A.    When the noncompliance is believed to be intentional and its effect on the financial statements is material

B.    If the effect of the noncompliance on the financial statements is material, whether intentional or unintentional

C.    When the effect of the noncompliance on the financial statements is believed to be both material and pervasive

D.    When the appropriate response by the entity includes reporting the noncompliance to parties outside the entity

A.

The auditor should, of course, communicate with those charged with governance matters involving noncompliance with laws and regulations that come to the auditor’s attention during the course of the audit, other than when the matters are clearly inconsequential; however, US GAAS only requires that the communi­cation take place as soon as practicable when the noncompliance is believed to be intentional and material. None of the other answer alternatives reflect the US GAAS requirement regarding timely communication with those charged with governance. Although the circumstance in incorrect answer D. is not a factor in determining if the communication to those charged with governance is required to be timely, if the auditor has identified or suspects noncompliance with laws and regulations, the auditor is required to determine whether the auditor has a responsibility to report the identified or suspected noncompliance to parties outside the entity.

When obtaining an understanding of the entity and its environment, the auditor should obtain an understanding of the entity's selection and application of accounting policies. In relation to this, the matter least likely to be considered is

A.    The methods the entity uses to account for significant and unusual transactions

B.    The financial reporting competencies of personnel involved in selecting and applying significant new or complex accounting standards

C.    Current and prospective financing requirements

D.    Financial reporting standards, and laws and regulations that are new to the entity and when and how the entity will adopt such requirements

C.

Current and prospective financing requirements is an example of a matter the auditor may consider when obtaining an understanding about the entity’s objectives and strategies and those related business risks that may result in risks of material misstatement; rather than a matter related to the entity’s selection and appli­cation of accounting policies, including the reasons for any changes (the auditor should evaluate whether the entity’s accounting policies are appropriate for its business and consistent with the applicable financial reporting framework and accounting policies used in the relevant industry).

When obtaining an understanding of the entity and its environment, the auditor should obtain an understanding of the nature of the entity. In relation to this, the matter least likely to be considered is

A.    The location of production facilities, warehouses, and offices and the location and quantities of inventories

B.    Planned or recently executed acquisitions or divestitures

C.    Beneficial owners and related parties

D.    Taxation

D.

Taxation is an example of a matter the auditor may consider when obtaining an understanding about external factors; rather than a matter related to the nature of the entity.

When obtaining an understanding of the entity and its environment, the auditor should obtain an understanding of the relevant industry, regulatory, and other external factors, including the applicable financial reporting framework. In relation to this, the matter least likely to be considered is

A.    Cyclical or seasonal activity

B.    Environmental requirements affecting the industry and the entity’s business

C.    Whether the entity has a complex structure

D.    Government policies currently affecting the conduct of the entity’s business

C.

Whether the entity has a complex structure is an example of a matter the auditor may consider when obtaining an understanding about the nature of the entity; rather than a matter related to relevant industry, regulatory, or other external factors, including the applicable financial reporting framework.

Editor’s note: Take a look at the answer choices in a bit more detail and you’ll notice that choice C. is an outlier from the four choices provided; choices A., B. and D. are all external/regulatory/industry factors, while having a complex structure is more client-focused and not necessarily an industry factor.

When obtaining an understanding of the entity and its environment, the auditor should obtain an understanding of the entity's objectives and strategies and those related business risks that may result in risks of material misstatement. In relation to this, the matter least likely to be considered is

A.    New products and services

B.    Expansion of the business

C.    Capital investment activities

D.    Use of IT

C.

Capital investment activities is an example of a matter the auditor may consider when obtaining an understanding about the nature of the entity; rather than a matter related to the entity’s objectives and strategies and those related business risks that may result in risks of material misstatement.

When performing analytical procedures in the planning stage,the auditor most likely would develop expectations by reviewing which of the following sources of information?

A.    Unaudited information from internal quarterly reports

B.    Various account assertions in the planning memorandum

C.    Comments in the prior year’s management letter

D.    The control risk assessment relating to specific financial assertions

A.

The purpose of analytics in the planning stage of the audit is to assist in planning the audit proce­dures that will be used to obtain audit evidence for specific account balances or classes of transactions. To accomplish this, one of the things the auditor should focus on is the transactions and events that have occurred since the last audit date. The unaudited information from internal quarterly reports would be the best source of information for developing expectations. Expectations, in this context, are the auditor’s predictions of recorded accounts or ratios. The effectiveness of analytical procedures depends on developing expectations that can reasonably be expected to identify unexpected relationships. Account assertions in the planning memorandum would not provide the needed data to develop such expectations. The auditor is interested in transactions and events that have occurred since the last audit date, so comments in the prior year’s management letter would have limited usefulness and, again, would probably not include data needed to derive expectations. Control risk affects the reliability of data and, thus, the precision of the expectation, but it would not be considered a source for developing expectations.

When should an auditor test an indirect control?

A.    When the control being tested is ineffective unless it is performed in conjunction with the indirect control

B.    When the control being tested depends upon the indirect control and the auditor determines it is also necessary to obtain audit evidence about the operating effectiveness of the indirect control

C.    When the control the auditor seeks to rely on is impractical to test and the test of the related indirect control will provide persuasive evidence that both controls are operating effectively

D.    When the control being tested and the related indirect control are over a significant risk

B.

An auditor should test an indirect control when the control being testing depends upon another control and the auditor determines it is also necessary to obtain audit evidence about the operating effectiveness of that control. Regarding incorrect answer a., it is not that the control being tested is necessarily ineffective without the support of the indirect control and thus the auditor must test both controls, but rather a case of the indirect control being of direct relevance to the auditor's test of the “main” control.

The other answer alternatives are false statements.

When testing financial statement disclosures, the most relevant assertions are (CVRU)

Completeness; Valuation, Allocation, and Accuracy; Rights and obligations, and occurrence; Understandability and classification.

When testing transactions, the most relevant assertions are (COVEU)

Completeness; Cutoff; Valuation, Allocation, and Accuracy; Existence and occurrence; Understandability and classification.

When the auditor identifies or suspects noncompliance with laws and regulations, the auditor should

A.    Consider withdrawing from the engagement.

B.    Discuss the matter with those charged with governance to obtain their guidance as to which level of management should be notified.

C.    Discuss the matter with the auditor’s legal counsel prior to notifying either management or those charged with governance.

D.    Obtain an understanding of the nature of the act; the circumstances in which it has occurred; and evaluate the possible effect on the financial statements.

D.

When the auditor identifies or suspects noncompliance with laws and regulations, the auditor should obtain an understanding of the nature of the act; the circumstances in which it has occurred; and obtain further information to evaluate the possible effect on the financial statements.

Regarding incorrect answer A., at this stage, the consideration of withdrawal would be premature. The auditor may consider whether withdrawal from the engagement is necessary when management or those charged with governance (1) do not take the reme­dial action that the auditor considers appropriate, even when the noncompliance is not material to the financial statements; or (2) refuse to accept a modified opinion on the financial statements; either when the noncom­pliance has a material effect on the financial statements and it has not been adequately reflected in the financial statements; or management or those charged with governance have limited the scope of the audit such that the auditor cannot make a determination about noncompliance. Under these circumstances, the auditor may with­draw from the engagement and indicate the reasons in writing to those charged with governance.

Incorrect answers B. and C. are false statements. If the auditor suspects noncompliance may exist, the auditor should discuss the matter with management (at a level above those involved with the suspected noncompliance, if pos­sible) and, when appropriate, those charged with governance. If management or, as appropriate, those charged with governance do not provide sufficient information that supports that the entity is in compliance with laws and regulations and the effect of the suspected noncompliance may be material to the financial statements, the auditor should consider the need to obtain legal advice.

When the user entity uses the services of a service organization

A.    The user auditor does not make reference to the service auditor’s report as a basis, in part, for the user auditor’s opinion on the user entity’s financial statements.

B.    If the user auditor makes reference to the service auditor, the service auditor’s name and signature should be included in the user auditor’s report.

C.    The user auditor should not refer to the work of a service auditor in the user auditor’s report containing a modified opinion.

D.    If the user auditor makes reference to the service auditor, the user auditor’s report should indicate that the portion of the audit and opinion related to the services of the service organization are the responsibility of the service auditor.

A.

The user auditor does not make reference to the service auditor’s report as a basis, in part, for the user auditor’s opinion on the user entity’s financial statements. The fact that a user entity uses a service organi­zation does not alter the user auditor’s responsibility to obtain sufficient appropriate audit evidence to afford a reasonable basis to support the user auditor’s opinion. However, when the user auditor expresses a modified opinion because of a modified opinion in a service auditor’s report, the user auditor is not precluded from refer­ring to the service auditor’s report if such reference assists in explaining the reason for the user auditor’s modi­fied opinion. The user auditor’s report should indicate that such reference to the service auditor does not diminish the user auditor’s responsibility for that opinion. In such circumstances, the user auditor need not iden­tify the service auditor by name and may need the consent of the service auditor before making such a reference.

Regarding incorrect answer B., the service auditor does not sign the user’s auditor’s report under any circumstances.

Regarding incorrect answer C., the user auditor should not refer to the work of a service auditor if the user auditor’s report contains an unmodified opinion. The only reason a reference should be made to the work of a service auditor is explained above in the explanation for the correct answer A.—a reason that does not apply if the user audit report contains an unmodified opinion.

Regarding incorrect answer D., the user auditor's report should not indicate shared responsibility with the service auditor under any circumstances.

Which explanation best describes how an auditor determines (as required by US GAAS) if a financial statement assertion is relevant for each significant class of transactions, account balance, and disclosure? A. By exercising professional judgment for the particular nature and characteristics of the entity B. By exercising professional skepticism, recognizing that circumstances may exist that cause the financial statements to be materially misstated C. By determining the source of likely potential misstatements in each significant class of transactions, account balance, and disclosure D. By performing walk-throughs of the related systems

C Identifying relevant assertions includes determining the source of likely potential misstatements in each significant class of transactions, account balance, and disclosure. Attributes indicating the potential rele­vance of an assertion include the nature of the assertion; volume of transactions or data related to the assertion; and the nature and complexity of the systems, including the use of information technology (IT), by which the entity processes and controls information supporting the assertion.

Which of audit risk components may be assessed in nonquantitative terms?

Control risk, detection risk and inherent risk.

Which of the board is responsible for establishing requirements for professional accountants for the International Federation of Accountants?

International Ethics Standards Board of Accountants. The International Ethics Standards Board for Accountants (IESBA) of the International Federation of Accountants (IFAC) issues the Code of Ethics for Professional Accountants.

Which of the following audit procedures would be most appropriate to test the valuation of the collateral of a delinquent loan receivable?

A.    Sending a positive confirmation letter to the debtor to confirm the loan balance.

B.    Performing a site visit to physically inspect the collateral.

C.    Reviewing the debtor's purchase records to test the historical value of the collateral.

D.    Obtaining a current value appraisal of the collateral.

The correct answer is (D).

Obtaining a current value appraisal of the collateral would be the most appropriate audit procedure to test the valuation of the collateral of a delinquent loan receivable. If the auditor is not competent enough to execute this task, he may engage a specialist to do the same.

(A) is incorrect because sending a positive confirmation letter to the debtor to confirm the loan balance will not reveal information about the collateral. It will only provide the loan balance.

(B) is incorrect because performing a site visit will only prove that the collateral exists, not the value of the collateral.

(C) is incorrect because a review of the debtor’s purchase records to the historical value of the collateral will not provide sufficient information about its current value.

Which of the following auditing procedures most likely would assist an auditor in identifying related party transactions? A. Inspecting correspondence with lawyers for evidence of unreported contingent liabilities; B. Vouching accounting records for recurring transactions recorded just after the balance sheet date; C. Reviewing confirmations of loans receivable and payable for indications of guarantees; D. Performing analytical procedures for indications of possible financial difficulties.

C. Reviewing confirmations of loans receivables and payable is useful for determining the existence of related party transactions because guarantees are commonly provided by or for related parties.

Which of the following best represents a key control for ensuring sales are properly authorized when assessing control risks for sales?

A.    The separation of duties between the billing department and the cash receipts approval department

B.    The use of an approved price list to determine unit selling price

C.    Copies of approved sales orders sent to the shipping, billing, and accounting departments

D.    Sales orders are sent to the credit department for approval

D.

Sending sales orders to the credit department for approval is a key control for ensuring that sales are properly authorized. Although the billing and cash receipts departments should be separate (to reduce the opportunity for any one person to both perpetrate and conceal fraud or errors in the normal course of their duties), their separation does not impact whether sales are properly authorized. Use of an approved price list is a control to ensure the accuracy of sales transactions rather than proper authorization. The receipt of approved sales orders by the shipping, billing, and accounting departments occurs after the fact.

Which of the following characteristics most likely would heighten an auditor's concern about the risk of material misstatements arising from fraudulent financial reporting? A. The entity's industry is experiencing declining customer demand. B. Employees who handle cash receipts are not bonded. C. Bank reconciliations usually include in-transit deposits. D. Equipment is often sold at a loss before being fully depreciated.

The correct answer is (A) Fraud is a deliberate act by one or more people, resulting in a misstatement of financial statements. It can be of two types: Fraudulent financial reporting (management fraud) and Misappropriation of assets (defalcation). A substantial decrease in client demand and an increase in business failures in either the sector or the economy as a whole provide management with an incentive (which is a fraud risk factor) to falsify financial statements resulting in fraudulent financial reporting and hence would heighten an auditor's concern about the risk of material misstatements arising from fraudulent financial reporting.

Which of the following circumstances most likely would cause an auditor to consider whether material misstatements exist in an entity's financial statements? A. Management places little emphasis on meeting earnings projections. B. The board of directors makes all major financing decisions. C. Significant deficiencies previously communicated to management are not corrected. D. Transactions selected for testing are not supported by proper documentation.

D. The auditor most likely would consider whether material misstatements exist when transactions selected for testing are not supported by proper documentation. Reduced emphasis on meeting earnings projections would be a factor decreasing the likelihood of earnings overstatements and assets (the most fre­quent misstatement). Having the board of directors make major financing decisions would decrease the incentive for management to use questionable reporting by reducing the amount of management’s responsibility. Signifi­cant deficiencies previously communicated to management may not have been corrected because of an unfa­vorable cost-benefit relationship.

Which of the following circumstances would an auditor most likely consider a risk factor relating to misstatements arising from fraudulent financial reporting? A. Several members of management have recently purchased additional shares of the entity's stock. B. Several members of the board of directors have recently sold shares of the entity's stock C. The entity distributes financial forecasts to financial analysts that predict conservative operating results. D. Management is interested in maintaining the entity's earnings trend by using aggressive accounting practices.

D. The fact that management is interested in maintaining the entity’s earnings trend by using aggres­sive accounting practices indicates that management may have an incentive to engage in fraudulent financial reporting also. Stock transactions of management or the board of directors are not ordinarily considered fraud risk factors. Aggressive financial forecasts as opposed to conservative financial forecasts would be considered a fraud risk factor. In other words, answer choices A. through C. are traditional practices by management/board of directors, while using aggressive accounting policies to maintain earnings would be a strong indicator of fraudulent financial reporting.

Which of the following components of internal control contributes most to a strong control environment?

A.    Controls are assessed through ongoing activities and evaluations.

B.    Management adheres to internal control policies.

C.    Duties are clearly defined and separated.

D.    Policy manuals provide a clear understanding of internal controls.

The correct answer is (B).

Management adhering to internal control policies is an important part of the control environment. Management reinforces expectations at the various levels of the organization, and it has a pervasive impact on the overall system of Internal Control and is the foundation for all other Internal Control components.

Controls being assessed through ongoing activities and evaluations are part of the “Monitoring” component of the internal control framework. Therefore,  (A) is incorrect.

Duties being clearly defined and separated are related to the segregation of duties, which are part of the “Control Activities” component of the internal control framework. Therefore, (C) is incorrect.

Policy Manuals providing a clear understanding of internal controls are part of the “Information and communication systems” component of the internal control framework. Therefore, (D) is incorrect.

Which of the following constitutes a potential risk associated with the use of information technology in an entity's internal control structure?

A.    A reduction in the ability to monitor the entity's activities.

B.    The facilitation of additional analyses.

C.    A reduction in the circumvention of controls.

D.    Unauthorized changes to systems.

The correct answer is (D).

Unauthorized changes to systems constitute a potential risk associated with the use of information technology in an entity’s internal control structure. Changes to information systems not approved are a big red sign of future risk exposure.

Use of information technology will not reduce the ability to monitor the entity's activities. The facilitation of additional analyses would help protect against risk, not expose it. A reduction in the circumvention of controls will help protect against potential risk as well.

Which of the following controls most likely would be effective in offsetting the tendency of sales personnel to maximize sales volume at the expense of high bad debt write-offs?

A.    Employees responsible for authorizing sales and bad debt write-offs are denied access to cash.

B.    Shipping documents and sales invoices are matched by an employee who does not have authority to write off bad debts.

C.    Employees involved in the credit-granting function are separated from the sales function.

D.    Subsidiary accounts receivable records are reconciled to the control account by an employee independent of the authorization of credit.

C.

The most effective control in offsetting the tendency of sales personnel to maximize sales volume at the expense of high bad debt write-offs would be the segregation of duties of those employees involved in credit-granting and sales functions. If this segregation exists, credit-granting employees should help to screen out those potential customers likely to result in high bad-debt write-offs.

Which of the following controls most likely would be effective in offsetting the tendency of sales personnel to maximize sales volume at the expense of high bad debt write-offs?

A.    Employees responsible for authorizing sales and bad debt write-offs are denied access to cash.

B.    Shipping documents and sales invoices are matched by an employee who does not have authority to write off bad debts.

C.    Employees involved in the credit-granting function are separated from the sales function.

D.    Subsidiary accounts receivable records are reconciled to the control account by an employee independent of the authorization of credit.

C.

The most effective control in offsetting the tendency of sales personnel to maximize sales volume at the expense of high bad debt write-offs would be the segregation of duties of those employees involved in credit-granting and sales functions. If this segregation exists, credit-granting employees should help to screen out those potential customers likely to result in high bad-debt write-offs.

Which of the following controls would a company most likely use to safeguard marketable securities when an independent trust agent is not employed?

A.    The investment committee of the board of directors periodically reviews the investment decisions delegated to the treasurer.

B.    Two company officials have joint control of marketable securities, which are kept in a bank safe-deposit box.

C.    The internal auditor and the controller independently trace all purchases and sales of marketable securities from the subsidiary ledgers to the general ledger.

D.    The chairman of the board verifies the marketable securities, which are kept in a bank safe-deposit box, each year on the balance sheet date.

B.

A bank safe-deposit box with two company officials in control of the assets provides strong physical control over the securities.

Answers A. and C. would not safeguard the physical marketable security.

Answer D. does not provide for dual control throughout the year, as is preferred.

Which of the following events most likely would indicate the existence of related party transactions?

A.    Ensuring the lives of key executives and listing the entity as beneficiary.

B.    Selling real estate at a price that differs significantly from its appraised value.

C.    Making a loan with specific scheduled terms for repayment of the funds.

D.    Granting stock options to key executives at favorable prices.

The correct answer is (B).

A related party is a person or an entity that is related to the reporting entity:  A person or a close member of that person's family is related to a reporting entity if that person has control, joint control or significant influence over the entity or is a member of its key management personnel.

Transactions that may be indicative of the existence of related parties are transactions that are different than the market value of transactions and include:

Borrowing or lending on zero (or unusually low / high) interest rates or with no scheduled terms for when or how the funds will be repaid.

Selling real estate at a price that differs significantly from its appraised value.

Exchanging property for similar property in a non-monetary transaction

Selling real estate at a price different from fair value would raise suspicion about the related party transactions. 

All other options take place in the ordinary course of business.

Which of the following factors is most relevant when an auditor considers the client’s organizational struc­ture in the context of control risk?

A.    Management’s attitude toward information processing and accounting departments

B.    The organization’s recruiting and hiring practices

C.    Physical proximity of the accounting function to upper management

D.    The suitability of the client’s lines of reporting

D.

Control risk is a function of the effectiveness of the design and operation of internal control in achiev­ing a company’s objectives relevant to the preparation of the financial statements. One of the five components of internal control is the control environment. One of the elements of the control environment is a company’s organizational structure. As it provides the framework for accomplishing the company’s objectives, a relevant organizational structure includes considering key areas of authority and responsibility and appropriate (suitable) lines of reporting. The key to selecting the best answer was to determine which was most related to a company’s organizational structure. Management’s attitude and recruiting and hiring practices are not directly related to organizational structure, as is the case with lines of reporting. The physical proximity of the accounting function to upper management could be completely unrelated to organizational structure.

Which of the following factors most likely would assist an external auditor in assessing the objectivity of the internal auditor?

A.    The organizational status of the director of internal audit

B.    The professional certifications of the internal audit staff

C.    The consistency of the internal audit reports with the results of work performed

D.    The appropriateness of internal audit conclusions in the circumstances

A.

When assessing the internal auditors’ objectivity, the external auditor may obtain information about such factors as the organizational status of the director of the internal audit function. The professional certifications of the internal audit staff is a factor to consider when assessing their competence; not their objectivity. Answers C. and D. are factors to consider when developing evaluation procedures for testing the effectiveness of internal auditors’ work which also provides information about their competence rather than their objectivity.

Editor’s note: An internal audit function reporting to the Audit Committee would be considered objective, versus an internal audit function designed to assist management.

which of the following factors most likely would heighten an auditors concern about the risk of fraudulent financial reporting

1. an overly complex organizational structure involving unusual lines of authority; 2. inability to generate positive cash flows from operations, while reporting large increases in earnings; 3. difficulty in determining the organization or individuals that have controlling interest in the entity; 4. high turnover of senior manage­ment, legal counsel, or those charged with governance.

Which of the following factors would least likely affect the extent of the auditor’s consideration of the cli­ent’s internal controls?

A.    The amount of time budgeted to complete the engagement

B.    The size and complexity of the client

C.    The nature of specific relevant controls

D.    The auditor’s prior experience with client operations

A.

The nature, extent, and timing of procedures the auditor chooses to perform in the consideration of internal control will vary depending on the size and complexity of the entity, previous experience with the entity, the nature of the specific controls used by the entity including the entity’s use of IT, the nature and extent of changes in systems and operations, and the nature of the entity’s documentation of specific controls.

Which of the following fraudulent activities most likely could be perpetrated due to the lack of effective internal controls in the revenue cycle?

A.    Fictitious transactions may be recorded that cause an understatement of revenues and an overstatement of receivables.

B.    Claims received from a customer for goods returned may be intentionally recorded in other customer accounts.

C.    Authorization of credit by personnel who receive cash may permit the misappropriation of cash.

D.    The failure to prepare shipping documents may cause an overstatement of inventory balances.

The correct answer is (C)

Recording of fictitious transactions would cause an overstatement of both revenues and receivables and not an understatement of revenues and an overstatement of receivables.

Claims received from a customer for goods returned are recorded in other customer accounts, it would not be a misstatement of the revenue or receivables and a fraud is not likely to be perpetrated. This would just be a misstatement of the related revenue or receivables individual component.

Failure to prepare shipping documents may cause an overstatement of inventory balances, the same is not likely to result in fraud.

The requirement is to identify the type of fraudulent activity which could most likely be perpetrated due to the lack of effective internal controls in the revenue cycle. Strong controls can be implemented in the revenue cycle by proper segregation of duties. The person who receives cash from customers (Custody) should not be authorizing credit (Authorization) to them. If these duties are done by the same person, they may collect cash from the entity's customers and not report it, resulting in the possibility of misappropriation of cash.

Which of the following fraudulent activities most likely could be perpetrated due to the lack of effective internal controls in the revenue cycle?

A.    Merchandise received is not promptly reconciled to the outstanding purchase order file.

B.    Obsolete items included in inventory balances are rarely reduced to the lower of cost or market value.

C.    The write-off of receivables by personnel who receive cash permits the misappropriation of cash.

D.    Fictitious transactions are recorded that cause an understatement of revenue and overstatement of receivables.

C.

Segregation of duties prevents an employee from committing fraud and subsequently concealing it. Proper segregation of duties separates the functions of record keeping, custody and authorization.

Answers A. and B. are not necessarily fraudulent activities. Fictitious transactions that overstate receivables also tend to overstate revenue, not understate it.

Which of the following fraudulent activities most likely could be perpetrated due to the lack of effective internal controls in the revenue cycle?

A.    Fictitious transactions may be recorded that cause an understatement of revenues and an overstatement of receivables.

B.    Claims received from customers for goods returned may be intentionally recorded in other customers' accounts.

C.    Authorization of credit memos by personnel who receive cash may permit the misappropriation of cash.

D.    The failure to prepare shipping documents may cause an overstatement of inventory balances.

C.

One of the most effective means of detecting or preventing fraud is the segregation of duties involving authorization, custody, and record keeping. Authorization of credit memos by personnel who receive cash (custody) is not segregation of these duties. The recording of fictitious transactions and failure to prepare shipping documents resulting in an overstatement of inventory likely would be discovered during reconciliations. Return claims applied to inappropriate accounts likely would be discovered by customers when monthly statements are issued.

Which of the following internal control procedures is not usually performed in the vouchers payable department?

A.    Matching the vendor's invoice with the related receiving report

B.    Approving vouchers for payment by having an authorized employee sign the vouchers

C.    Indicating the asset and expense accounts to be debited

D.    Accounting for unused prenumbered purchase orders and receiving reports

D.

Accounting for unused purchase orders and receiving reports by the same department does not provide sufficient segregation of duties for the authorization, custody, and reporting functions of an effective internal control system. The lack of segregation of these items could result in an employee defalcation scheme.

Answers A. – C. each represent a procedure that is usually performed in the vouchers payable department.

Which of the following internal control procedures most likely would deter lapping of collections from customers?

A.    Independent internal verification of dates of entry in the cash receipts journal with dates of daily cash summaries

B.    Authorization of write-offs of uncollectible accounts by a supervisor independent of credit approval

C.    Segregation of duties between receiving cash and posting the accounts receivable ledger

D.    Supervisory comparison of the daily cash summary with the sum of the cash receipts journal entries

C.

The segregation of custody and reporting is an important internal control that is most likely to deter a lapping scheme. Authorization of write-offs by a supervisor is a good control but would not in itself prevent an employee from misappropriating accounts receivable collections because the supervisor is unfamiliar with the accounts, as indicated in the question, and would, therefore, not be aware of customers who are delinquent in their payments. Answers a. and d. are examples of good internal controls but would not by themselves uncover a lapping scheme.

Which of the following is a control procedure that most likely could help prevent employee payroll fraud?

A.    The personnel department promptly sends employee termination notices to the payroll supervisor.

B.    Employees who distribute payroll checks forward unclaimed payroll checks to the absent employees' supervisors.

C.    Salary rates resulting from new hires are approved by the payroll supervisor.

D.    Total hours used for determination of gross pay are calculated by the payroll supervisor.

A.

By promptly notifying the payroll supervisor of employee terminations, the personnel department would avoid a terminated employee from continuing to be paid. Unclaimed payroll checks should be forwarded to the Treasurer’s office. Salary rates should be controlled in the personnel office. Total hours should be deter­mined and approved prior to getting to the payroll department.

Although answers C. and D. are good controls, they would not prevent employee payroll fraud.

Which of the following is a control procedure that most likely could help prevent employee payroll fraud?

A.    The personnel department promptly sends employee termination notices to the payroll supervisor.

B.    Employees who distribute payroll checks forward unclaimed payroll checks to the absent employees' supervisors.

C.    Salary rates resulting from new hires are approved by the payroll supervisor.

D.    Total hours used for determination of gross pay are calculated by the payroll supervisor.

A.

By promptly notifying the payroll supervisor of employee terminations, the personnel department would avoid a terminated employee from continuing to be paid. Unclaimed payroll checks should be forwarded to the Treasurer’s office. Salary rates should be controlled in the personnel office. Total hours should be deter­mined and approved prior to getting to the payroll department.

Although answers C. and D. are good controls, they would not prevent employee payroll fraud.

Which of the following is a definition of control risk?

A.    The risk that a material misstatement will not be prevented or detected on a timely basis by the client’s internal controls.

B.    The risk that the auditor will not detect a material misstatement.

C.    The risk that the auditor's assessment of internal controls will be at less than the maximum level.

D.    The susceptibility of material misstatement assuming there are no related internal controls, policies, or procedures.

A.

Control risk is the risk that a misstatement that could occur in an assertion about a class of transaction, account balance, or disclosure and that could be material, either individually or when aggregated with other misstatements, will not be prevented, or detected and corrected, on a timely basis by the entity's internal control.

The risk that the auditor will not detect a material misstatement refers to detection risk, not control risk.

The risk that the auditor's assessment of internal controls will be at less than the maximum level is not a risk—the use of risk assessment procedures and tests of controls may appropriately result in such a conclusion.

The susceptibility of material misstatement assuming there are no related internal controls, policies, or procedures refers to inherent risk, not control risk.

Which of the following is an example of an inherent risk that an auditor should consider? A. Technological developments that may render inventory obsolete B. Posting of unauthorized journal entries C. An incorrect formula in a worksheet used to calculate a LIFO inventory reserve D. Inaccurate physical inventory count

The correct answer is (A). Inherent Risk (IR) is Susceptibility of a relevant assertion to a material misstatement. Factors in the entity and its environment influence IR. E.g., If entity’s inventory becomes obsolete rapidly due to technology, IR ↑ for inventory If entity lacks sufficient working capital, IR ↑ for several assertions If entity in a declining industry with many business failures, IR ↑ for several assertions Technological developments that may render inventory obsolete is an example of an inherent risk that an auditor should always consider.

Which of the following is an inherent limitation of internal controls?

A.    Judgmental sampling

B.    Collusion

C.    Segregation of duties

D.    Employee peer reviews

B.

Collusion of two or more people to circumvent controls is an inherent limitation of any system of internal control. An internal control system, no matter how well designed and operated, cannot give an entity absolute assurance that its objectives will be met because any system can contain inherent limitations. Other inherent limitations of internal control are faulty human judgment in decision making, human errors that cause breakdowns in internal control, and inappropriate management override of controls.

Regarding incorrect answer A., judgmental sampling, is part of the application of an auditing procedure; not an inherent limitation of internal control.

Incorrect answers C. and D., segregation of [incompatible] duties and employee peer reviews, respectively, are examples of controls; not inherent limitations.

Which of the following is most likely to be a response to the assessed risk of material misstatement at the assertion level? A. Assigning more experienced individuals or additional individuals with specialized skill and knowledge to the engagement B. Evaluating whether the selection and application of accounting policies by the entity is indicative of management bias C. Incorporating an element of unpredictability in the selection of the nature, timing, and extent of audit procedures D. Inspecting certain assets

D. The physical observation and/or inspection of assets are examples of procedures performed in response to the assessed risk of material misstatement at the assertion level, i.e., the assertion of existence. The other answer alternatives are overall responses to address the assessed risks of material misstatement due to fraud at the financial statement level rather than the assertion level (the assertion level is at a more specific level of testing than the financial statement level and hence a more thorough procedure is required)

Which of the following is the best way to compensate for the lack of adequate segregation of duties in a small organization?

A.    Disclosing lack of segregation of duties to the external auditors during the annual review

B.    Replacing personnel every three or four years

C.    Requiring accountants to pass a yearly background check

D.    Allowing for greater management oversight of incompatible activities

Answer D., the best way to compensate for the lack of adequate segregation of duties in a small organization is to allow for greater management oversight of incompatible activities because it is the more constant control and a practical solution.

Regarding incorrect answer A., disclosing lack of segregation of [incompatible] duties to the external auditors during the annual review would not be effective because it could occur after a problem had occurred. Moreover, establishing and maintaining effective internal control is the responsibility of management, NOT the auditors.

Regarding incorrect answer B., replacing personnel every three or four years is not effective because, again, it could occur after the fact, not to mention it is so draconian it could actually motivate some individuals to commit fraud if they became aware of the policy.

Regarding incorrect answer C., an issue discovered during a yearly background check of accountants is not as effective because it could also occur after the fact. Also, even if the background check did not reveal any issues, collusion could occur with individuals who had not committed fraud in the past because they were subjected to pressures they had not experienced before.

Which of the following is the earliest determination to be made by an auditor for a group audit performed in accordance with US GAAS?

A.    The materiality for the group financial statements as a whole

B.    Whether to act as the auditor of the group financial statements

C.    Which components are significant components

D.    Whether to reference a component auditor in the audit report on the group financial statements

B.

One of the first determinations to be made by an auditor for a group audit is whether to act as the auditor of the group financial statements. Although the other answer alternatives are matters that are determined early in the audit, they are made by an auditor who has decided to act as auditor for the group financial statements. Relevant factors in determining whether to act as the auditor of the group financial statements include, among other things, the following: (1) the individual financial significant of the components for which the auditor of the group financial statements will be assuming responsibility; (2) the extent to which significant risks of material misstatement of the group financial statements are included in the components for which the auditor of the group financial statements will be assuming responsibility; and (3) the extent of the group engagement team’s knowledge of the overall financial statements.

Editor’s note: Don’t let the term “group” trick you on questions like these; no matter whether group or component, big or small, the earliest thing you should be doing as an auditor is to consider whether to perform the audit. You already know and understand the due diligence required prior to accepting an engagement, and these same items apply with group/component audits.

Which of the following most accurately describes the process of a walkthrough?

A.    Testing and documenting the results of tests of selected controls.

B.    Inspection of selected documents, records, and internal control documentation.

C.    Observation of an entity's activities and operations.

D.    Following a transaction from its origination until it is reflected in the financial statements.

The correct answer is (D).

A walkthrough is a risk assessment procedure to obtain evidence about the design and implementation of internal control. Here the auditor traces transactions from inception through recording in books to reporting in financial statements.

(A) is incorrect because testing and documenting the results of tests of controls happens subsequent to risk assessment procedures like walkthroughs.

(B) and (C) are incorrect because an inspection of documents and observation of activities are individual components of the process of a walkthrough, which involves walking through from inception to financial reporting. Other than inspection and observation, a walkthrough also focuses on inquiry of client personnel and reperformance of some of their activities.

Which of the following most accurately describes the process of a walkthrough?

A.    Testing and documenting the results of tests of selected controls.

B.    Inspection of selected documents, records, and internal control documentation.

C.    Observation of an entity's activities and operations.

D.    Following a transaction from its origination until it is reflected in the financial statements.

The correct answer is (D).

A walkthrough is a risk assessment procedure to obtain evidence about the design and implementation of internal control. Here the auditor traces transactions from inception through recording in books to reporting in financial statements.

(A) is incorrect because testing and documenting the results of tests of controls happens subsequent to risk assessment procedures like walkthroughs.

(B) and (C) are incorrect because an inspection of documents and observation of activities are individual components of the process of a walkthrough, which involves walking through from inception to financial reporting. Other than inspection and observation, a walkthrough also focuses on inquiry of client personnel and reperformance of some of their activities.

Which of the following most likely would be an internal control procedure designed to detect errors and fraud concerning the custody of inventory?

A.    Periodic reconciliation of work in process with job cost sheets

B.    Segregation of functions between general accounting and cost accounting

C.    Independent comparisons of finished goods records with counts of goods on hand

D.    Approval of inventory journal entries by the storekeeper

C.

An independent comparison of finished goods records with counts of goods on hand is designed to detect errors and fraud concerning inventory custody as it provides an independent reconciliation of the two amounts.

Answers A. and B. do not consider the inventory itself nor inventory custody. The storekeeper should not be able to approve inventory journal entries because he or she has custody of the assets.

Which of the following most likely would not be considered an inherent limitation of the potential effectiveness of an entity's internal control?

A.    Incompatible duties

B.    Management override

C.    Mistakes in judgment

D.    Collusion among employees

A.

While incompatible duties can be segregated and therefore controlled, the possibility of manage­ment override and collusion among employees to circumvent controls will still exist. Mistakes in judgment also cannot be controlled.

Which of the following procedures most likely could assist an auditor in identifying related-party transac­tions?

A.    Performing tests of controls concerning the segregation of duties

B.    Evaluating the reasonableness of management’s accounting estimates

C.    Reviewing confirmations of compensating balance arrangements

D.    Scanning the accounting records for recurring transactions

C.

Business structure and operating style occasionally are designed deliberately to obscure relation­ships. Because compensating balance (补偿性余额) arrangements and non monetary transactions obscure the form of trans­actions,they are suspect. Compensating balance arrangements involve related parties more often than the other answer options. Segregation of duties applies whether the parties to a transaction are related or not. Reason­able and unreasonable accounting estimates both exist with and without related parties present. Entities fre­quently have recurring transactions with independent entities.

Which of the following procedures most likely would not be an internal control procedure designed to reduce the risk of errors in the billing process?

A.    Comparing control totals for shipping documents with corresponding totals for sales invoices

B.    Using computer programmed controls on the pricing and mathematical accuracy of sales invoices

C.    Matching shipping documents with approved sales orders before invoice preparation

D.    Reconciling the control totals for sales invoices with the accounts receivable subsidiary ledger

D.

Answers A. – C. are all controls applicable to the billing process.

Answer D. is a control used after the billing process.

Which of the following procedures would a CPA most likely perform in the planning stage of a financial statement audit?

A.    Obtain representations from management regarding the availability of all financial records

B.    Communicate with the audit committee concerning the prior year's audit adjustments

C.    Make inquiries of the client's attorney regarding pending and threatened litigation and assessments

D.    Compare recorded financial information with anticipated results from budgets and forecasts

D.

The auditor is required to perform analytics in the planning stage of the audit. These procedures assist the auditor in planning the nature, extent, and timing of substantive tests; thus, they should focus on enhancing the auditor's understanding of the client's business and the transactions and events that have occurred since the last audit and identifying areas that may represent specific risks relevant to the audit. Analytics involve comparisons of recorded amounts or ratios developed from recorded amounts to expectations developed by the auditor. Comparison of recorded financial information with anticipated results from budgets and forecasts is appropriate for this planning stage. Representations from management are dated the same date as the auditor's report. Communication regarding the prior year's audit adjustments typically occurs near the end of the audit for that year. Inquiry of the client's attorney tends to occur after the planning stage, as part of the substantive tests.

Which of the following procedures would an auditor most likely include in developing the overall audit strategy of a financial statement audit?

A.    Obtaining a representation letter from the client

B.    Examining documents to detect noncompliance with laws and regulations having a material effect on the financial statements

C.    Considering whether the client's accounting estimates are reasonable in the circumstances

D.    Determining the extent of involvement of the client's internal auditors

D.

Of the procedures listed, an auditor is most likely to determine the extent of involvement of the client's internal auditors in developing the overall audit strategy of a financial statement audit. A representation letter from the client's management (and, when appropriate, those charged with governance) is obtained at the end of an audit, not the beginning—it should be as of the date of the audit report on the financial statements. Examining documents and considering the reasonableness of estimates are procedures performed during the audit.

Which of the following procedures would an auditor most likely perform in planning a financial statement audit?

A.    Inquiring of the client's legal counsel concerning pending litigation

B.    Comparing the financial statements to anticipated results

C.    Searching for unauthorized transactions that may aid in detecting unrecorded liabilities

D.    Examining computer generated exception reports to verify the effectiveness of internal controls

B.

Analytical procedures, such as comparing the financial statements to anticipated results, are used as risk assessment procedures during the planning phase of an audit to provide the basis for planning further audit procedures.

Inquiry of the client’s legal counsel about pending litigation is an audit procedure that would be performed near the end of the engagement. (It is preferable that the legal counsel’s response be as close to the date of the audit report as is practicable to avoid the need to obtain updated information.) Even if the inquiry (as opposed to the date of the response) occurs earlier in the audit, it is unlikely to be made during the planning (risk assessment) phase. A related activity, i.e., obtaining a general understanding of the legal and regulatory framework applicable to the entity and how the entity is complying with that framework, is actually best performed very early in the planning phase (prior to the auditor’s identification and assessment of the risks of material misstatement), but this is different than the substantive audit procedure known as the letter of inquiry.

Answer C. is a substantive audit procedure that would be performed during the audit engagement to support management's assertion of completeness.

When performing risk assessment procedures to obtain an understanding of controls that are relevant to the audit, the auditor evaluates the design of those controls and determines whether they have been implemented. This is different than answer D. which is an example of testing the operating effectiveness of a control. Testing the operating effectiveness of a control is a substantive procedure (tests of controls) performed during the audit as a response to the auditor’s assessment of the risks of material misstatement.

Editor Note: Analytical procedures are also used as substantive procedures during the audit to detect material misstatements at the assertion level and near the end of the audit to assist the auditor when forming an overall conclusion on the financial statements.

Which of the following procedures would an auditor most likely perform to test controls relating to management's assertion about the completeness of cash receipts for cash sales at a retail outlet?

A.    Observe the consistency of the employees' use of cash registers and tapes

B.    Inquire about employees' access to recorded but undeposited cash

C.    Trace the deposits in the cash receipts journal to the cash balance in the general ledger

D.    Compare the cash balance in the general ledger with the bank confirmation request

A.

Assertions about completeness are tested by testing whether or not all cash is recorded. If employ­ees consistently use cash registers and tapes, it is likely that all cash is recorded. Inquiry about employees’ access to undeposited recorded cash, comparing the cash receipts journal to the general ledger, and comparing the cash balance in the general ledger with the bank confirmation request, only test assertions about recorded cash.

Which of the following procedures would an auditor most likely include in establishing the overall audit strategy of a financial statement audit?

A.    Obtain an understanding of the entity's risk assessment process

B.    Identify specific internal control activities designed to prevent fraud

C.    Evaluate the reasonableness of the entity's accounting estimates

D.    Perform cutoff tests of the entity's sales and purchases

A.

The entity's risk assessment process is part of its internal control. Gaining an understanding of the entity's internal control is the first of these four procedures to be performed in an audit and is an example of a matter considered in establishing the overall audit strategy. The auditor identifies specific activities designed to prevent fraud when considering relying on internal control. The other two options are substantive procedures performed during the audit.

Which of the following questions would an auditor least likely include on an internal control questionnaire concerning the initiation and execution of equipment transactions?

A.    Are requests for major repairs approved at a higher level than the department initiating the request?

B.    Are prenumbered purchase orders used for equipment and periodically accounted for?

C.    Are requests for purchases of equipment reviewed for consideration of soliciting competitive bids?

D.    Are procedures in place to monitor and properly restrict access to equipment?

D.

The procedures in place to monitor and properly restrict access to equipment would more likely be observed by the auditor than be part of a questionnaire, as this is very important.

Answer A. represents the authorization function of the control structure, which would be included on the internal control questionnaire.

Answer B. represents a procedure to test the accounting and recording function of the control structure, which the auditor would include in an internal control questionnaire. Requests for competitive bids reduce the possibil­ity of an individual’s personal gain at the expense of the business in unusual and material transactions.

Which of the following questions would an auditor least likely include on an internal control questionnaire concerning the initiation and execution of equipment transactions?

A.    Are requests for major repairs approved at a higher level than the department initiating the request?

B.    Are prenumbered purchase orders used for equipment and periodically accounted for?

C.    Are requests for purchases of equipment reviewed for consideration of soliciting competitive bids?

D.    Are procedures in place to monitor and properly restrict access to equipment?

D.

The procedures in place to monitor and properly restrict access to equipment would more likely be observed by the auditor than be part of a questionnaire, as this is very important.

Answer A. represents the authorization function of the control structure, which would be included on the internal control questionnaire.

Answer B. represents a procedure to test the accounting and recording function of the control structure, which the auditor would include in an internal control questionnaire. Requests for competitive bids reduce the possibil­ity of an individual’s personal gain at the expense of the business in unusual and material transactions.

Which of the following questions would most likely be included in an internal control questionnaire concerning the completeness assertion for purchases?

A.    Is an authorized purchase order required before the receiving department can accept a shipment or the vouchers payable department can record a voucher?

B.    Are purchase requisitions prenumbered and independently matched with vendor invoices?

C.    Is the unpaid voucher file periodically reconciled with inventory records by an employee who does not have access to purchase requisitions?

D.    Are purchase orders, receiving reports, and vouchers prenumbered and periodically accounted for?

D.

Assertions about completeness deal with whether all transactions and accounts that should be presented in the financial statements are so included. One step in assuring this is the periodic reconciliation of prenumbered purchase orders, receiving reports, and vouchers.

Which of the following questions would most likely be included in an internal control questionnaire concerning the completeness assertion for purchases?

A.    Is an authorized purchase order required before the receiving department can accept a shipment or the vouchers payable department can record a voucher?

B.    Are purchase requisitions prenumbered and independently matched with vendor invoices?

C.    Is the unpaid voucher file periodically reconciled with inventory records by an employee who does not have access to purchase requisitions?

D.    Are purchase orders, receiving reports, and vouchers prenumbered and periodically accounted for?

D.

Assertions about completeness deal with whether all transactions and accounts that should be presented in the financial statements are so included. One step in assuring this is the periodic reconciliation of prenumbered purchase orders, receiving reports, and vouchers.

Which of the following represents an inherent limitation of internal controls?

A.    Bank reconciliations are not performed on a timely basis.

B.    The CEO can request a check with no purchase order.

C.    Customer credit checks are not performed.

D.    Shipping documents are not matched to sales invoices.

B.

One of the inherent limitations of internal control is the inappropriate management override of controls. An example of this would be if the CEO can request a check with no purchase order. The other answer alternatives describe weaknesses in internal control, but they are not inherent limitations. Inherent limitations are intrinsic—the incorrect answers are examples of failures in internal control that can be remedied.

Which of the following services would constitute a management function under Government Auditing Standards, and result in the impairment of a CPA's independence if performed by the CPA? A. Developing entity program policies B. Providing methodologies, such as practice guides C. Providing accounting opinions to a legislative body D. Recommending internal control procedures

A. In general, any situation that requires the application of judgment to make a decision tends to impair independence. Developing client policies is listed as a general activity that impairs auditor independence. Providing or recommending methodologies or internal control procedures does not impair independence; client management still bears the burden of deciding to select and implement those methodologies or procedures. Providing accounting opinions to a legislative body or other group generally does not impair independence.

Which of the following situations most likely could lead to an embezzlement scheme?

A. The accounts receivable bookkeeper receives a list of payments prepared by the cashier and per-sonally makes entries in the customers’ accounts receivable subsidiary ledger. B. Each vendor invoice is matched with the related purchase order and receiving report by the vouchers payable bookkeeper who personally approves the voucher for payment. C. Access to blank checks and signature plates is restricted to the cash disbursements bookkeeper who personally reconciles the monthly bank statement. D. Vouchers and supporting documentation are examined and then canceled by the treasurer who personally mails the checks to vendors.

C.

Access to blank checks and signature plates is restricted to the cash disbursements bookkeeper who personally reconciles the monthly bank statement is the situation that most likely could lead to an embezzle­ment scheme. Bank reconciliations should not be prepared by persons who sign checks or keep the records for receipts or disbursements. This is an example of incompatible functions which place a person in the position to both perpetrate and conceal errors or fraud in the normal course of their duties. The duties of authorization, record keeping, and custody of assets should be separated. Additionally, bank reconciliations should not be done by persons with responsibility for handling cash receipts. The other answer alternatives are examples of appropriate duties/control practices.

Which of the following situations most likely represents the highest risk of a misstatement arising from the misappropriation of assets?

A. A large number of bearer bonds on hand B. A large number of inventory items with low sales prices C. A large number of transactions processed in a short period of time D. A large number of fixed assets with easily identifiable serial numbers

A. A large number of bearer bonds on hand most likely represents the highest risk of a misstatement arising from the misappropriation of assets. This is a risk factor categorized as an opportunity. Certain charac­teristics or circumstances may increase the susceptibility of assets to misappropriation, for example, large amounts of cash on hand or processed; inventory items that are small in size, of high value, or in high demand; easily convertible assets, such as bearer bonds, diamonds, or computer chips; or fixed assets that are small in size, marketable, or lacking observable identification of ownership.

Which of the following situations represents a risk factor that relates to misstatements arising from misappropriation of assets? A. High turnover of senior management B. A lack of independent checks C. A strained relationship between management and the current or predecessor auditor D. An inability to generate cash flows from operations while reporting earnings and earnings growth Answer b., a lack of independent checks (or inadequate segregation of duties) is one of the risk factors related to misstatements arising from the misappropriation of assets.

while reporting earnings and earnings growth Answer b., a lack of independent checks (or inadequate segregation of duties) is one of the risk factors related to misstatements arising from the misappropriation of assets. The other answer alternatives are risk factors related to misstatements arising from fraudulent financial reporting rather than the misappropriation of assets. Editor Note: There are two types of fraud relevant to the auditor's consideration—fraudulent financial reporting and misappropriation of assets.

Which of the following statements correctly describes the "top-down approach" used during an audit of internal control over financial reporting?

A.    Begin reviewing balance sheet accounts and then review income statement accounts.

B.    Begin reviewing income statement accounts and then review balance sheet accounts.

C.    Begin by understanding the overall risks to internal control over financial reporting at the financial statement level.

D.    Begin by understanding the overall risks to internal control over financial reporting at the general ledger level.

The correct answer is (C).

A top-down approach begins at the financial statement level and with the auditor's understanding of the overall risks to ICFR. The auditor then focuses on entity-level controls and works down to significant accounts and disclosures and their relevant assertions.

(A) and (B) are incorrect because before reviewing balance sheet or income statement accounts, the auditor should understand overall risks to ICFR at the financial statement level.

(D) is incorrect because the auditor should begin by understanding the overall risks to ICFR at the financial statement level and then move down to accounts at the general ledger level.

Which of the following statements indicates that the auditor has gained a sufficient understanding of a client's internal controls related to the sales order process?

A.    The auditor noted in a narrative that the documentation for the sales order system showed the printing of a shipment-exception report listing non-invoiced shipments.

B.    In a statistically valid sample of 100 sales transactions, the auditor found five undiscovered exceptions and concluded that the system was weak.

C.    The auditor interviewed the company's supervisor of sales clerks and reviewed six shipment-exception reports that were randomly selected and that showed significant unrecorded balances.

D.    The auditor compared sales orders processed to processing clerk headcount for three years and noted that processed orders significantly declined while clerk headcount remained the same.

The correct answer is (A).

Noting in a narrative that the documentation for the sales order system showed the printing of a shipment-exception report listing non-invoiced shipments, is an indicator that the auditor has gained a sufficient understanding of the internal controls related to the sales order process. During the “Understanding the entity and its processes” stage of the audit, the auditor usually performs tests of Design on controls.

To understand and document system processes and internal controls, the auditor uses procedures such as talking to the client, internal control and internal control evaluation questionnaires, narrative notes and flowcharts.

Hence, noting in a narrative clearly implies that the auditor has gained a sufficient understanding of a client's internal controls.

(B), (C) and (D) relate to tests of the operating effectiveness of key controls.

Which of the following statements is correct concerning analytical procedures used in planning an audit engagement?

A.    They often replace the tests of controls.

B.    They may use financial and nonfinancial data aggregated at a high level.

C.    They often involve the comparison of assertions developed by management to ratios calculated by an auditor.

D.    They are often used to develop an auditor's preliminary judgment about materiality.

B.

Analytical procedures performed as risk assessment procedures, i.e., analytical procedures used to plan the audit, often use data aggregated at a high level. The objective is to identify the existence of unusual transactions or events and amounts, ratios, and trends that might indicate matters that have financial statement and audit planning ramifications.

Analytical procedures are not used to replace tests of controls. Analytical procedures can be used as substantive tests to obtain audit evidence about particular assertions (which is not part of the planning stage).

Substantive analytical procedures often involve the comparison of expectations developed by the auditor (not assertions developed by management) to recorded amounts (or ratios developed from recorded amounts) to achieve an audit objective related to a particular assertion made by management.

The determination of materiality is made early in the planning process. A percentage is often applied to a chosen benchmark as a starting point in determining materiality for the financial statements as a whole. However, this type of analysis is not the same as performing analytical procedures in the context of an audit. (Analytical procedures are defined by US GAAS as evaluations of financial information through analysis of plausible relationships among both financial and nonfinancial data. Analytical procedures also encompass such investigation, as is necessary, of identified fluctuations or relationships that are inconsistent with other relevant information or that differ from expected values by a significant amount.)

Which of the following statements is correct regarding internal control?

A.    A well-designed internal control environment ensures the achievement of an entity's control objectives.

B.    An inherent limitation to internal control is the fact that controls can be circumvented by management override.

C.    A well-designed and operated internal control environment should detect collusion perpetrated by two people.

D.    Internal control is a necessary business function and should be designed and operated to detect all errors and fraud.

B.

An inherent limitation to internal control is the fact that controls can be circumvented by management override. Other limitations inherent to internal control include human failures such a faulty decision making and simple errors or mistakes. Additionally, controls can be circumvented by the collusion of two or more people. A well-designed internal control system does not ensure the achievement of an entity's control objectives nor can it be designed and operated to detect all errors and fraud; it only provides the entity with reasonable, not absolute, assurance due to these inherent limitations.

Which of the following statements is correct with respect to fraud encountered during an audit engagement of a non-issuer? A. The distinguishing factor between fraud and error is the materiality of the transaction involved. B. An auditor who initially detects fraud ultimately makes the legal determination of whether fraud has actually occurred. C. Fraudulent financial reporting can include the unintentional misstatement of amounts or disclosures in financial statements. D. It is often difficult to detect fraudulent intent in matters involving accounting estimates and the application of accounting principles.

The correct answer is (D). Fraud is an intentional act that results in a material misstatement in financial statements. Such intent is often difficult to determine, particularly in matters involving accounting estimates and the application of accounting principles. For example, unreasonable accounting estimates may be unintentional or may be the result of an intentional attempt to misstate the financial statements. Hence,It is often difficult to detect fraudulent intent in matters involving accounting estimates and the application of accounting principles and estimates lead to a high degree of judgment and subjectivity on part of management. (A) is incorrect because the distinguishing factor between fraud and error is not defined by materiality. (B) is incorrect because an auditor detects and assesses the implications of fraud encountered during an audit engagement but does not have legal authority to make any further decisions. CPAs are not attorneys. (C) is incorrect because unintentional misstatements are not included in fraudulent financial reporting.

Which of the following would an auditor most likely use in determining materiality for the financial statements as a whole?

A.    The anticipated sample size of the planned substantive tests

B.    The entity's annualized interim financial statements

C.    The results of the internal control questionnaire

D.    The contents of the representation letter

B.

The auditor would most likely use the entity's annualized interim financial statements. Although applying a percentage to a chosen benchmark, such as a category of reported income, as a starting point to determine materiality for the financial statements as a whole is not required, it is a method that is often used. With regard to the chosen benchmark, relevant financial data ordinarily includes prior periods’ financial results and financial positions; the period-to-date financial results and financial position; and budgets or forecasts for the current period. The auditor should take into account any significant changes in the entity's circumstances and relevant changes in the economy or the industry in which the entity operates. The anticipated sample size of the planned substantive tests and the internal control questionnaire are, if anything, dependent on materiality, not vice versa.

The representation letter is dated as the date of the audit report and thus its content would not be available during the planning stage of the audit.

Which statement regarding the auditor's risk assessment of material misstatement due to fraud is not true? A. It should be ongoing throughout the audit B. It should be based on the presumption that risks of fraud exist in revenue recognition C. Assessed risks due to fraud should all be treated by the auditor as significant risks D. The auditor should evaluate whether each assessed risk due to fraud is a significant risk

D. There is no need for the auditor to evaluate whether each assessed risk due to fraud is a significant risk; thus, answer D. is the correct answer, i.e., the only statement that is not true. The auditor should treat all of them as significant risks (answer C.) and, accordingly, obtain an understanding of the entity’s related controls, including the evaluation of whether such controls have been suitably designed and implemented to mitigate such fraud risks. Answers A., and B. are also true statements. Regarding answer A., the auditor’s risk assess­ment of material misstatement due to fraud should be ongoing throughout the audit, following the initial assessment. Regarding answer B., when identifying and assessing the risks of material misstatement (RMM) due to fraud, the auditor should, based on a presumption that risks of fraud exist in revenue recognition, evaluate which types of revenue, revenue transactions, or assertions give rise to such risks. (If the auditor has concluded that this presumption is overcome in the circumstances of the engagement, the auditor should include in the audit documentation the reasons for that conclusion.)

While performing a test of details during an audit, an auditor determined that the sample results supported the conclusion that the recorded account balance was materially misstated. It was, in fact, not materially misstated. This situation illustrates the risk of:

Why do the risks of material misstatement need to be identified and assessed at the financial statement level in addition to the relevant assertion level for classes of transactions, account balances, and disclosures?

A. Risks at the financial statement level require an overall response by the auditor.

B. The audit opinion is based on the financial statement as a whole.

C. When establishing the overall audit strategy, the auditor should determine materiality for the financial statements as a whole.

D. Misstatements in the financial statements can arise from either fraud or error.

A. The nature of risks at the financial statement level requires an overall response by the auditor. All of the other answer alternatives are true, but they are not responsive to the question.

Why does GAAS establish two categories of laws and regulations?

A.    GAAS requires a different auditing approach for each category because the auditor has different responsibilities for each.

B.    One category affects the determination of amounts in the financial statements while the other may only affect the financial statement disclosures.

C.    The auditor is required to have the professional competence to determine whether an act constitutes noncompliance in one of the categories, but is not expected to be able to make such a legal determination with regard to the other.

D.    Different materiality considerations are applied to one category vs. the other.

A.

GAAS requires a different auditing approach for each category because the auditor has different responsibilities for each, as follows:
 

For the provisions of those laws and regulations generally recognized to have a direct effect on the determination of material amounts and disclosures in the financial statements, the auditor's responsibility is the same as that for misstatements caused by fraud or error—to obtain sufficient appropriate audit evidence regarding material amounts and disclosures in the financial statements that are determined by the provisions of those laws and regulations.

For the other category, that does not have such a direct effect; the auditor's responsibility is limited to performing specified audit procedures that may identify noncompliance with those laws and regulations that may have a material effect on the financial statements.

The other answer alternatives are false statements. In regard to answer c., whether an act assignable to either GAAS category constitutes noncompliance with laws and regulations is a matter for legal determination, which ordinarily is beyond the auditor's professional competence to determine. Nevertheless, the auditor's training, experience, and understanding of the entity and its industry or sector may provide a basis to recognize that some acts coming to the auditor's attention may constitute noncompliance with laws and regulations.